You are on page 1of 102

Ropa Arienza-Arpilleda

ENRIQUEZ vs. SUN LIFE ASSURANCE CO. OF CANADA


41 PHIL 269 / November 29, 1920

FACTS:

On September 24, 1917, Joaquin Herrer made application to the Sun


Life Assurance Company of Canada through its office in Manila for a life annuity and
paid the sum of P6,000, two days later to the manager of the
company’s Manila office and was given a receipt. On November 26, 1917, the head
office of the company at Montreal, Canada. Gave notice of acceptance by cable to
Manila.

On December 18, 1917, Atty.Aurelio A. Torres wrote to the Manila office of the
company stating that Herrer desired to withdraw his application. The following day the
local office replied to Mr. Torres, stating that the policy had been issued, and called
attention to the notification of November 26, 1917. This letter was received by Mr.
Torres on the morning of December 21, 1917. Mr. Herrer died on December 20, 1917.

On trial,the chief clerk of the Manila office testified that he prepared the letter and
handed it to the local manager, Mr. E. E. White, for signature. The local manager
testified to having received the cablegram accepting the application of Mr. Herrer from
the home office on November 26, 1917 and that on the same day he signed a letter
notifying Mr. Herrer of this acceptance. The witness further said that letters, after being
signed, were sent to the chief clerk and placed on the mailing desk for transmission. For
the defense, Atty. Manuel Torres testified to having prepared the will of Joaquin Ma.
Herrer. That on this occasion, Mr. Herrer mentioned his application for a life annuity, and
that he said that the only document relating to the transaction in his
possession was the provisional receipt. Rafael Enriquez, the administrator
of the estate, testified that he had gone through the effects of the deceased and
had found no letter of notification from the insurance company to Mr. Herrer.

ISSUE:

Whether or not Herrer received notice of acceptance of his application to hold


that the contract for a life annuity was perfected.

RULING:

NO. The contract for a life annuity in the case at bar was not perfected because it
has not been proved satisfactorily that the acceptance of the application ever came to
the knowledge of the applicant. The letter of 26 November 1917, notifying Mr. Ferrer
that his application had been accepted, was prepared and signed in the local office of
the insurance company, was placed in the ordinary channels for transmission, but was
never actually mailed and thus was never received by the applicant.

In addition, the provisional receipt provides three things had to be accomplished


by the insurance company before there was a contract: (1) There had to be a medical
examination of the applicant; (2) there had to be approval of the application by the head
office of the company; and (3) this approval had in some way
to be communicated by the company to the applicant.
The law applicable to the case is found to be the second paragraph of article
1262 of the Civil Code providing that an acceptance made by letter shall not bind
the person making the offer except from the time it came to his knowledge.

The law of insurance is consequently now found in the Insurance Act and the
Civil Code. While the Insurance Act deals with life insurance, it is silent as to the
methods to be followed in order that there may be a contract of insurance. On the other
hand, the Civil Code, in article 1802, not only describes a contact of life annuity
markedly similar to the one herein considered, but in two other articles, gives strong
clues as to the proper disposition of the case. For instance, article 16 of the Civil Code
provides that "In matters which are governed by special laws, any deficiency of the
latter shall be supplied by the provisions of this Code."

On the supposition that the special law on the subject of insurance is deficient in
enunciating the principles governing acceptance, the subject-matter of the Civil code, if
there be any, would be controlling. In the Civil Code is found article 1262 providing that
"Consent is shown by the concurrence of offer and acceptance with respect to the thing
and the consideration which are to constitute the contract. An acceptance made by
letter shall not bind the person making the offer except from the time it came to his
knowledge.

In this case, judgment was reversed, and the plaintiff recovered from the defendant the
sum of P6,000 with legal interest.
Silhouette B. Adobas

CONSTANTINO vs. ASIA LIFE INSURANCE CO.


G.R. No. L-1669 / August 31, 1950

FACTS:

In consideration of the sum of P176.04 as annual premium duly paid to it, the Asia Life
Insurance Company (a foreign corporation incorporated under the laws of Delaware, U.S.A.),
issued on September 27, 1941, its Policy No. 93912 for P3,000, whereby it insured the life of
Arcadio Constantino for a term of twenty years. The first premium covered the period up to
September 26, 1942. The plaintiff Paz Lopez de Constantino was regularly appointed
beneficiary. The policy contained these stipulations, among others:

This POLICY OF INSURANCE is issued in consideration of the written and printed


application here for a copy of which is attached hereto and is hereby made a part hereof,
and of the payment in advance during the lifetime and good health of the Insured of the
annual premium of One Hundred fifty-eight and 4/100 pesos Philippine currency 1 and of
the payment of a like amount upon each twenty-seventh day of September
hereafter during the term of Twenty years or until the prior death of the Insured.
(Emphasis supplied.)
xxx xxx xxx
All premium payments are due in advance and any unpunctuality in making any such
payment shall cause this policy to lapse unless and except as kept in force by the Grace
Period condition or under Option 4 below. (Grace of 31 days.)

After that first payment, no further premiums were paid. The insured died on September
22, 1944.

It is admitted that the defendant, being an American corporation , had to close its branch
office in Manila by reason of the Japanese occupation, i.e. from January 2, 1942, until the year
1945.

ISSUE:

Had the policies lapsed for non-payment of premiums?

RULING:

Yes, the policies had lapsed. The United States rule declares that the contract is
not merely suspended, but is abrogated by reason of non-payments is peculiarly of the
essence of the contract. It additionally holds that it would be unjust to allow the insurer
to retain the reserve value of the policy, which is the excess of the premiums paid over
the actual risk carried during the years when the policy had been in force. This rule was
announced in the well-known Statham 6case which, in the opinion of Professor Vance, is
the correct rule. After perusing the Insurance Act, we are firmly persuaded that the non-
payment of premiums is such a vital defense of insurance companies that since the very
beginning, said Act no. 2427 expressly preserved it, by providing that after the policy
shall have been in force for two years, it shall become incontestable (i.e. the insurer
shall have no defense) except for fraud, non-payment of premiums, and military or
naval service in time of war (sec. 184 [b], Insurance Act). And when Congress recently
amended this section (Rep. Act No. 171), the defense of fraud was eliminated, while the
defense of nonpayment of premiums was preserved. Thus the fundamental character of
the undertaking to pay premiums and the high importance of the defense of non-
payment thereof, was specifically recognized.
Kenneth B. Minglana

PEREZ vs. COURT OF APPEALS


January 28, 2000

FACTS:

The deceased Primitivo Perez, husband of herein petitioner, was originally


insured for 20,000 with BF Life Insurance. He was convinced by an agent of BF Life to
apply for additional coverage worth 50,000. He then filed an application form together
with the rest of the requirements and paid the corresponding premium to the agent. One
important condition for perfection stated in the application was that the insured must
receive and accept the policy while he is in good health. Prior to the transmittal of his
records by the agent to the main office of BF Life, Primitivo met an accident and died.
Not knowing of his death, the agent forwarded his application form to the main office
and a corresponding insurance was issued in his name. As beneficiary, Virginia filed a
claim for the proceeds of the policy worth 50,000. The claim was denied by BF Life on
the ground that the policy on which it is based, though issued, was not actually
perfected since the death of the insured made it impossible for the latter to be notified of
its acceptance hence, there is no meeting of minds with respect to that contract. Virginia
counters that the condition that her husband should receive and accept the insurance
policy while in good health is potestative and therefore void.

ISSUE:

Was there a perfected contract of insurance between Primitivo and BF Life?

RULING:

None. When the applicant died, his application papers were still in the branch office and
were still subject to the approval of BF Life. It matters not that an insurance policy was
wrongfully issued since it is impossible for him to receive and accept the same, as conditioned in
his application, because at the time of its issuance, he is already dead. A contract of insurance,
like all other contracts, must be assented to by both parties, either in person or through their
agents and so long as an application for insurance has not been either accepted or rejected, it is
merely a proposal or an offer to make a contract.
Sundae June A. Jugao

TY vs. FIRST NATIONAL SURETY & ASSURANCE CO. INC.


1 SCRA 1324

FACTS:

The plaintiff herein Diosdado C. Ty was employed as operator mechanic


foreman in the Broadway Cotton Factory. He insured himself in 18 local
insurance companies, which issued to him personal accident which
stipulated, among others, that for partial disability resulting to the loss of
either hand, the insurer shall be liable for P650.00. It was further stated in
the policies that, “That loss of a hand shall mean the loss by amputation
through the bones of the wrist." Plaintiff's beneficiary was his employer,
Broadway Cotton Factory, which paid the insurance premiums. On December
24, 1953, a fire broke out which totally destroyed the Broadway Cotton
Factory. Fighting his way out of the factory, plaintiff was injured on the left
hand by a heavy object. He underwent medical treatment. As a result, Ty
suffered a temporary total disability of his left hand which prevented him
from performing his work or labor necessary in the pursuance of his
occupation. Plaintiff filed the corresponding notice of accident and notice of
claim with the defendants to recover indemnity but it was rejected by the
latter claiming that such disability was not covered by the policy.

ISSUE:

Is Ty entitled to indemnity under the insurance policy?

RULING:

No. The court cannot go beyond the clear and express conditions of the
insurance policies, all of which define partial disability as loss of either hand
by amputation through the bones of the wrist." There was no such
amputation in the case at bar. The agreement contained in the insurance
policies is the law between the parties. As the terms of the policies are clear,
express and specific that only amputation of the left hand should be
considered as a loss thereof, an interpretation that would include the mere
fracture or other temporary disability not covered by the policies would
certainly be unwarranted.
Junyvil B. Tumbaga

SIMON DE LA CRUZ vs. THE CAPITAL INSURANCE AND SURETY CO.,


INC.
G.R. No. L-21574, (17 SCRA 599)

FACTS:

Eduardo de la Cruz, the son of herein petitioner, was the holder of an


accident insurance policy. In connection with the celebration of the New Year,
the insured, a non-professional boxer, participated in a boxing contest. In the
course of his bout with another person, likewise a non-professional, of the
same height, weight, and size, Eduardo slipped and was hit by his opponent
on the left part of the back of the head, causing Eduardo to fall, with his head
hitting the rope of the ring. The insured died with the cause of death
reported as “hemorrhage intercranial, left”. The insurer refused to pay the
proceeds of the policy on the ground that the death of the insured, caused by
his participation in a boxing contest, was not accidental and, therefore, not
covered by insurance.

ISSUE:

Whether or not the death of the insured is covered by the policy.

RULING:

Yes. The terms “accident” and “accidental” as used in the insurance


contract, have not acquired any technical meaning, and are construed by the
courts in their ordinary and common acceptation. Thus, the terms have been
taken to mean that which happen by chance or fortuitously, without intention
and design, and which is unexpected, unusual, and unforeseen. An accident
is an event that proceeds from an unknown cause and, therefore, not
expected. Without the unintentional slipping of the deceased, perhaps he
would not have received the blow in the head and would not have died.
Boxing is attended with some risks of external injuries, but any injury
received in the course of the game could be accidental. In boxing, as in other
equally physically rigorous sports, such as basketball or baseball, death is
not ordinarily anticipated to result. If, therefore, it ever does, the injury or
death can only be accidental or produced by some unforeseen happening or
event as what occurred in this case. The insurer was liable.
William Z. Radaza

PANATON vs. MALAYAN INSURANCE CO.


2 Court of Appeals Report 78

FACTS:

A personal accident policy was issued by Malayan Insurance Co. to Panaton


covering “loss of legs” which was defined in the policy as the amputation of the legs.
The insured met an accident resulting in the total paralysis of both legs. When Panaton
claimed recovery from the insurance, the insurer refused to pay, because there was no
loss of legs since the legs of the insured were not amputated. The insured claimed that
the total paralysis of both legs should be considered as loss of legs even if they were
not amputated, and therefore covered by the insurance.

ISSUE:

Whether or not permanent and total paralysis of both legs should be considered
as equivalent to loss of legs to make the insurer liable under the insurance policy.

RULING:

The insurer is liable because “loss of legs” should be interpreted as to include the
permanent and total paralysis of both legs. The interpretation of the term “loss of legs”
as limited to amputation of both legs to the exclusion of permanent and total paralysis of
both legs would be contrary to public good, sound morality and public policy. It would
cause a desperate man to cause an amputation to be performed since his legs are of no
use for life, in order to avail of the benefits of the policy. The permanent and total
paralysis of both legs suffered by the insured was equivalent to loss of both legs, since
he will obviously be bedridden for the rest of his life.
Aurora Luanne R. Cembrano

FORTUNE INSURANCE AND SURETY CO., INC. vs. CA


244 SCRA 308

FACTS:

An armored car owned by private respondent Producers Bank of the Philippines was
robbed while transferring cash. It then sought reimbursement from petitioner under a theft or
robbery insurance policy. Petitioner refused contending that since the assigned armored car
driver and security guard took part in the robbery, the cause of the loss was excluded from the
coverage of the insurance policy. Under the General Exceptions Clause, Fortune is not liable in
case of loss caused by any dishonest, fraudulent or criminal act of any employee or authorized
representative of the insured.
Producers Bank opposes the contention stating that the driver and the security guard are not its
employees nor authorized representatives at the time of the robbery. Having been offered by their
respective agencies (PRC Management and Unicorn Security), they were merely an assigned
armored car driver and security guard.

ISSUE:

Whether or not the assigned armored car driver and the security guard of Producers Bank
qualify as its employee or representative, and if so, whether or not Fortune Insurance is liable
under the insurance policy.

RULING:

The assigned driver and security guard qualify as representatives of Producers Bank. The
Supreme Court held that the term employee as used in the policy must be generally and
universally understood. Thus, the assigned driver and security guard were, in respect of
transferring Producer’s money, are its authorized representatives. Producers entrusted the two
with the specific duty to safely transfer the money.
Insofar as Fortune is concerned, it was its intention to exclude and exempt from
protection and coverage losses arising from dishonest, fraudulent, or criminal acts of persons
granted or having unrestricted access to Producers' money or payroll. Hence, Fortune is exempt
from liability under the general exceptions clause of the insurance policy.

Christine Mae P. Navarra

WHITE GOLD MARINE SERVICES, INC. vs. PIONEER INSURANCE AND


SURETY CORPORATION AND THE STEAMSHIP MUTUAL
UNDERWRITING ASSOCIATION (BERMUDA) LTD.
464 SCRA 448 / July 28, 2005

FACTS:

Petitioner White Gold Marine procured a protection and indemnity


coverage for its vessels from Steamship Mutual through Pioneer Insurance.
White Gold was issued a Certificate of Entry and Acceptance and Pioneer
then also issued receipts evidencing payments for the coverage. When White
Gold failed to fully pay its accounts, Steamship Mutual refused to renew the
coverage and thereafter filed a case for collection of sum of money to
recover the unpaid balance. White Gold on the other hand, filed a complaint
before the Insurance Commission claiming that Steamship Mutual violated
Sections 186and 187 of the Insurance Code, while Pioneer violated Sections
299,300 and 301 in relation to Sections 302 and 303, thereof.

The Insurance Commission dismissed the complaint. It said that there


was no need for Steamship Mutual to secure a license because it was not
engaged in the insurance business. It explained that Steamship Mutual was a
Protection and Indemnity Club (P & I Club). Likewise, Pioneer need not obtain
another license as insurance agent and/or a broker for Steamship Mutual
because the latter was not engaged in the insurance business. The Court of
Appeals affirmed the decision of the Insurance Commissioner. It held that
Pioneer merely acted as a collection agent of Steamship Mutual.

ISSUES:

1. Whether or not Steamship Mutual, a P & I Club, engaged in the


insurance business in the Philippines.

2. Whether or not Pioneer needs a license as an insurance agent/broker


for Steamship Mutual.

RULING:

1. The test to determine if a contract is an insurance contract or not,


depends on the nature of the promise, the act required to be
performed, and the exact nature of the agreement in the light of the
occurrence, contingency, or circumstances under which the
performance becomes requisite. It is not by what it is called.

A P & I Club is "a form of insurance against third party liability, where
the third party is anyone other than the P & I Club and the members."
By definition then, Steamship Mutual as a P & I Club is a mutual
insurance association engaged in the marine insurance business.

The records reveal Steamship Mutual is doing business in the country


albeit without the requisite certificate of authority mandated by
Section 187 of the Insurance Code. It maintains a resident agent in the
Philippines to solicit insurance and to collect payments in its behalf. We
note that Steamship Mutual even renewed its P & I Club cover until it
was cancelled due to non-payment of the calls. Thus, to continue doing
business here, Steamship Mutual or through its agent Pioneer, must
secure a license from the Insurance Commission. Since a contract of
insurance involves public interest, regulation by the State is necessary.
Thus, no insurer or insurance company is allowed to engage in the
insurance business without a license or a certificate of authority from
the Insurance Commission.

2. Pioneer is the resident agent of Steamship Mutual as evidenced by the


certificate of registration issued by the Insurance Commission. It has
been licensed to do or transact insurance business by virtue of the
certificate of authority issued by the same agency. However, a
Certification from the Commission states that Pioneer does not have a
separate license to be an agent/broker of Steamship Mutual.

Although Pioneer is already licensed as an insurance company, it needs


a separate license to act as insurance agent for Steamship Mutual as
provided in Section 299 of the Insurance Code.
Albert G. Cong

BLUE CROSS HEALTH CARE vs. OLIVARES


600 SCRA 413 (2009)

FACTS:

Neomi Olivares applied for a health care program with Blue Cross Health Care
(Blue Cross) for the amount of P12,117.00. In the health care agreement, ailments due
to “pre-existing conditions” were excluded from the coverage. Barely thirty eight days
after, she suffered from a stroke. She was later confined in Medical City and because of
Blue Cross’ refusal to pay due to her lack of medical certification, she was constrained
to pay the bill of P34,217.20 and was discharged. Olivares filed suit in the MTC.

The health care company rebutted by saying that the physician didn’t disclose
the condition due to the patient’s invocation of the doctor-client privilege. The MTC
dismissed the case for a lack of cause of action because of such non-disclosure. On
appeal, the RTC, reversed the ruling of MTC, awarded the amount of the hospital bills
plus P60,000.00 in damages and held that it was the burden of petitioner to prove that
the stroke of Olivares was excluded from the coverage of the health care program for
being caused by a pre-existing condition.

ISSUE:

Was Blue Cross able to prove that Olivares’ stroke was caused by a pre-existing
condition and therefore excluded from the coverage of the health care agreement?

RULING:

No. In Philamcare Health Systems vs. CA, 429 Phil. 82 (2002), it held that a
health care agreement is in the nature of a non-life insurance. It is an established rule in
insurance contracts that when their terms contain limitations on liability, they should be
construed strictly against the insurer. These are contracts of adhesion the terms of
which must be interpreted and enforced stringently against the insurer which prepared
the contract. This doctrine is equally applicable to health care agreements.

Blue Cross never presented any evidence to prove that Olivares’ stroke
was due to a pre-existing condition. It merely speculated that the doctor’s
report would be adverse to her, based on her invocation of the doctor-patient
privilege. This was a disputable presumption at best. And since Blue Cross
had the burden of proving exception to liability, it should have made its own
assessment of whether Olivares had a pre-existing condition when it failed to
obtain the attending physician's report. The mere reliance on a disputable
presumption does not meet the strict standard required under our
jurisprudence.

Rogaciano A. Quico III

PHILIPPINE HEALTH CARE PROVIDERS, INC. vs. CIR


GR No. 167330 Sep 18, 2009

FACTS:

Philippine Health Care Providers, Inc. is a domestic corporation primarily


engaged in the business of providing prepaid group practice health care delivery
system. On January 27, 2000, the Commissioner of Internal Revenue sent an
assessment letter to the petitioner informing it and demanding payment of P224, 702,
614. 18 in back taxes, surcharge, and interests. The deficiency is composed mostly of
unpaid documentary stamp tax (DST) imposed on the petitioner’s agreement with its
members.
Petitioner protested before the CIR but due to the latter’s inaction; it filed a
petition for review before the Court of Tax Appeals. The CTA rendered a decision
partially granting the petition for review. The petitioner was ordered to pay P53M instead
of the original P225M. Furthermore, the CIR was ordered to desist from collecting DST
tax. Respondent CIR appealed the decision before the Court of Appeals. According to
him, the petitioner’s healthcare agreement is a contract of insurance and as such, is
subject to DST under Section 185 of the 1997 Tax Code. The CA rendered a decision
reversing the earlier decision of the CTA. It ordered the petitioner to pay P123M in DST.

Petitioner appealed the decision before the Supreme Court which affirmed the
CA’s decision. The SC held that the petioner’s health care agreement during the
pertinent period was in the nature of non-life insurance which is a contact of indemnity.
The Court further ruled that contracts between companies like petitioner and its
beneficiaries under their plans are treated as insurance contract. The petitioner filed a
motion for reconsideration.
ISSUE:

Whether or not the health care agreement between petitioner and its
beneficiaries is an insurance contract.

RULING:

The Supreme Court ruled in favor of the petitioner and granted the motion for
reconsideration. The Court ruled that the health care agreement between the
petitioner’s and its beneficiaries is not a contract of insurance.
The Court based its decision on the fact that the HMO agreement does not
qualify as an insurance business based on the “principal object and purpose test.” The
test is based on Section 2 (2) of the Insurance Code. Accordingly, an enterprise is
considered engaged in an insurance business when the principal object of the
enterprise is the assumption of risk and the indemnification of loss. If the enterprise
assumes risk and indemnifies beneficiaries for losses, then it is an insurance company.

American courts have pointed out that the main difference between an HMO and
an insurance company is that HMOs undertake to provide or arrange for the provision of
medical services through participating physicians while insurance companies simply
undertake to indemnify the insured for medical expenses incurred up to a pre-agreed
limit.

A substantial portion of petitioner’s services covers preventive and diagnostic


medical services intended to keep members from developing medical conditions or
diseases. As an HMO, it is its obligation to maintain the good health of its
members. Accordingly, its health care programs are designed to prevent or to minimize
the possibility of any assumption of risk on its part. Thus, its undertaking under its
agreements is not to indemnify its members against any loss or damage arising from a
medical condition but, on the contrary, to provide the health and medical services
needed to prevent such loss or damage.

Overall, petitioner appears to provide insurance-type benefits to its members


(with respect to its curative medical services), but these are incidental to the principal
activity of providing them medical care. The “insurance-like” aspect of petitioner’s
business is miniscule compared to its noninsurance activities. Therefore, since it
substantially provides health care services rather than insurance services, it cannot be
considered as being in the insurance business.

Lastly, it is significant that petitioner, as an HMO, is not part of the insurance


industry. This is evident from the fact that it is not supervised by the Insurance
Commission but by the Department of Health. In fact, in a letter dated September 3,
2000, the Insurance Commissioner confirmed that petitioner is not engaged in the
insurance business. This determination of the commissioner must be accorded great
weight. It is well-settled that the interpretation of an administrative agency which is
tasked to implement a statute is accorded great respect and ordinarily controls the
interpretation of laws by the courts.

Mary Christine Anthonette M. Salise-Punzalan


SERRANO vs. CA
G.R. No. L-35529 July 16, 1984

FACTS:

On or about January 1, 1965, upon application of the SYSTEM, Group Mortgage


Redemption Policy No. GMR-1 was issued by Private Life Insurance Companies operating in the
Philippines for a group life insurance policy on the lives of housing loan mortgagors of the
SYSTEM. Under this Group Mortgage Redemption scheme, a grantee of a housing loan of the
SYSTEM is required to mortgage the house constructed out of the loan and the lot on which it
stands. The SYSTEM takes a life insurance on the eligible mortgagor to the extent of the
mortgage indebtedness such that if the mortgagor dies, the proceeds of his life insurance under
the Group Redemption Policy will be used to pay his indebtedness to the SYSTEM and the
deceased's heirs will thereby be relieved of the burden of paying for the amortization of the
deceased's still unpaid loan to the SYSTEM (p. 25, rec.).

Petitioner herein is the widow of the late Bernardo G. Serrano, who, at the time of his
death, was an airline pilot of Air Manila, Inc. and as such was a member of the Social Security
System.

On November 10, 1967, the SYSTEM approved the real estate mortgage loan of the late
Bernardo G. Serrano for P37,400.00 for the construction of the applicant's house (pp. 25-26,
rec.).

On December 26, 1967, a partial release in the amount of P35,400.00 was effected and
devoted to the construction of the house (p. 2, rec.). As a consequence, a mortgage contract was
executed in favor of the SYSTEM by the late Captain Serrano with his wife as co-mortgagor.

On March 8, 1968, Captain Serrano died in a plane crash and because of his death, the
SYSTEM closed his housing loan account to the released amount of P35,400.00 (p. 26, rec.).
On December 2, 1968, the petitioner sent a letter addressed to the Chairman of the Social
Security Commission requesting that the benefits of the Group Mortgage Redemption Insurance
be extended to her.

The letter of the petitioner was referred to the Administrator of the SYSTEM, who
recommended its disapproval on the ground that the late Captain Serrano was not yet covered by
the Group Mortgage Redemption Insurance policy at the time of his death on March 8, 1968. In
its resolution No. 1365 dated December 24, 1968, the Social Security Commission sustained the
said stand of the SYSTEM and thereby formally denied the request of the petitioner (p. 26, rec.).

ISSUE:

The only issue to be resolved is the correctness of the interpretation given by the
respondent Commission which was upheld by the respondent Court as to the applicability of the
Mortgage Redemption Insurance plan particularly on when coverage on the life of the mortgagor
commences.
RULING:

The ambiguity in Section 3 of Article II should be resolved in favor of the petitioner.


"The interpretation of obscure words or stipulations in a contract shall not favor the party who
caused the obscurity" (Article 1377, Civil Code). WE have held that provisions, conditions or
exceptions tending to work a forfeiture of insurance policies should be construed most strongly
against those for whose benefit they are inserted, and most favorably toward those against whom
they are intended to operate (Trinidad vs. Orient Protective Ass., 67 Phil. 181).

The problem manifests itself in Sections 2 and 3 of the same article of the Group
Mortgage Redemption Insurance Policy. Section 2 provides that "any mortgagor who is eligible
for coverage on or after the Date of Issue shall be automatically insured, ..." (emphasis
supplied); while Section 3 provides that the insurance "shall take effect from the beginning of the
amortization period of such Mortgage loan or partialrelease of Mortgage Loan " (emphasis
supplied).

Section 2 of Article II of the Group Mortgage Redemption Insurance Policy provides that
insurance coverage shall be "automatic" and limited only by the amount of insurance and age
requirement. While the same section has for its title the mode of acceptance, what is controlling
is the meaning of the provision itself. The said section can only convey the Idea that the
mortgagor who is eligible for coverage on or after the date of issue shall be automatically
insured. The only condition is that the age requirement should be satisfied, which had been
complied with by the deceased mortgagor in the instant case.

Under said Section 2, mortgage redemption insurance is not just automatic; it is


compulsory for all qualified borrowers. This is the same automatic redemption insurance applied
to all qualified borrowers by the GSIS (Government Service Insurance System) and the DBP
(Development Bank of the Philippines). Indeed, the Mortgage Redemption Insurance Policy of
the GSIS provides:

Sec. 2. ... This policy is granted subject to the terms and conditions set forth at the
back hereof and in consideration of the application therefor and shall take effect
on the date of the first date of the aforementioned loan (p. 126, CA rec.; emphasis
supplied).

WE take judicial notice of the Mortgage Contract being issued by the Social Security
System in connection with applications for housing loans, specifically Section 16 thereof:

Section 16. — (a) The loan shall be secured against the death of the borrower
through the Mortgage Redemption Insurance Plan; (b) Coverage shall take effect
on the date of the first release voucher of the loan and shall continue until the real
estate mortgage loan is fully paid; ... (emphasis supplied)
Ropa Arienza- Arpilleda

MALAYAN INSURANCE CO., INC. vs. CRUZ ARNALDO and PINCA


154 SCRA 672 October 12, 1987

FACTS:

On June 7, 1981, Malayan Insurance Co. (MICO),issued fire insurance for the
amount of P14,000 on the property of private respondent, Pinca, effective July 1981-
1982. MICO later allegedly cancelled the policy for non-payment of the premium and
sent a notice to Pinca. On Dec. 24 Adora, an agent of MICO received Pinca’s payment,
which was remitted to MICO.

On Jan. 18, 1982, Pinca’s property was completely burned. On Feb. 5, MICO
returned Pinca’s payment to Adora on the ground that her policy had been cancelled;
the latter refused to accept it. Her demand for payment having been rejected by MICO,
Pinca went to the Insurance Commission. Public respondent Arnaldo,
the Insurance Commissioner, sustained Pinca, hence this petition from MICO.

ISSUES:

1. WON there was a valid insurance contract at the time of the loss.
2. WON Adora was authorized to receive such payment.

RULING:

Issue No. 1.

YES, there was a a valid insurance contract at the time of loss. MICO's view that
there was no existing insurance at the time of the loss sustained by Pinca because her
policy never became effective for non-payment of premium is untenable.
Petitioner relies heavily on Sec 77 of the Insurance Code to contest this, the
said provision requiring payment of premium as soon as the thing is exposed to the peril
insured against and that the policy is invalid without it. However, this is not applicable
in the instant case as payment was eventually made. Payment was in fact made,
rendering the policy operative as of June 22, 1981, and removing it from the provisions
of Article 77, Thereafter, the policy could be cancelled on any of the supervening
grounds enumerated in Article 64 (except "nonpayment of premium") provided the
cancellation was made in accordance therewith and with Article 65. Section 64 reads as
follows:

SEC. 64. No policy of insurance other than life shall be cancelled by the insurer except
upon prior notice thereof to the insured, and no notice of cancellation shall be effective
unless it is based on the occurrence, after the effective date of the policy, of one or more
of the following:

(a) non-payment of premium;


(b) conviction of a crime arising out of acts increasing the hazard insured against;
(c) discovery of fraud or material misrepresentation;
(d) discovery of willful, or reckless acts or commissions increasing the hazard
insured against;
(e) physical changes in the property insured which result in the property
becoming uninsurable; or
(f) a determination by the Commissioner that the continuation of the policy would
violate or would place the insurer in violation of this Code.

As for the method of cancellation, Section 65 provides that a valid cancellation


must, therefore, require concurrence of the following conditions:

(1) There must be prior notice of cancellation to the insured;


(2) The notice must be based on the occurrence, after the effective date of the
policy, of one or more of the grounds mentioned;
(3) The notice must be (a) in writing, (b) mailed, or delivered to the named
insured, (c) at the address shown in the policy;
(4) It must state (a) which of the grounds mentioned in Section 64 is relied upon
and (b) that upon written request of the insured, the insurer will furnish the facts
on which the cancellation is based.

The Court also finds it strange that MICO only sought to return Pinca’s Jan.15
payment only on Feb. 5, long after her house had burned down—this makes petitioner’s
motives highly suspicious.

MICO claims to have sent a notice to Pinca, who flatly denied receiving one.
Pinca did not have to prove this since the strict language of Sec. 64 requires that
MICO ensure the cancellation was actually sent to and received by the insured.
Payment was thus legally made on the original transaction and validly received
by Adora, who was not informed of the alleged cancellation and thus saw no reason
to reject the payment.

Issue No. 2

YES, Adora was authorized to receive payment by MICO. Sec. 306 of the
Insurance Code provides that any insurance company that delivers a policy to its agent
is deemed to have authorized such agent to receive payment of premium on its behalf.
It is a well-known principle under the law of agency that payment to an authorized agent
is equivalent to payment to the principal himself. MICO’s acknowledgement of Adora as
its agent thus defeats its contention that he was not authorized to receive payments on
its behalf.
Silhouette B. Adobas

LUMIBAO vs. INTERMEDIATE APPELLATE COURT


189 SCRA 469, 13 Sept 1990

FACTS:

Petitioner, Nora Lumibao, is a life insurance underwriter or agent and a member of a


group of insurance underwriters known as Bescon Insurance Agencies, Inc., representing the
Manila Bankers Life Insurance Corporation. The petitioner had induced private respondent to
take out a life insurance policy from Manila Bankers Life Insurance Corporation by promising
him a rebate equivalent to 50% of the first annual premium payment.

ISSUE:

May the court compel the petitioner to pay the rebate to Eugenio Trinidad?

RULING:

No, the Court may not. The agreement entered into between the parties was void
for being contrary to the provisions of Pres. Decree No. 612 [otherwise known as the
Insurance Code] and public policy.

Section 361 of Pres. Decree No. 612 states:

No insurance company doing business in the Philippines or any agent thereof, no


insurance broker, and no employee or other representative of any such insurance
company, agent, or broker, shall make, procure or negotiate any contract of insurance or
agreement as to policy contract, other than is plainly expressed in the policy or other
written contract issued or to be issued as evidence thereof, or shall directly or shall
indirectly, by giving or sharing a commission or in any manner whatsoever, pay or
allow or offer to pay or allow to the insured or to any employee of such insured, either as
an inducement to the making of such insurance or after such insurance has been effected,
any rebate from the premium which is specified in the policy, or any special favor or
advantage in the dividends or other benefits to accrue thereon, or shall give or offer to
give any valuable consideration or inducement of any kind, directly or indirectly, which
is not specified in such policy or contract of insurance; nor shall any such company, or
any agent thereof, as to any policy or contract of insurance issued, make any
discrimination against any Filipino in the sense that he is given less advantageous rates,
dividends or other policy conditions or privileges than are accorded to other nationals
because of his race

The agreement between the parties consists of an undertaking on the part of


private respondent to take out a life insurance policy with Manila Bankers Life Insurance
Corporation and, on the part of petitioner, to give private respondent a rebate on the first
premium payment. There is no indication on record that petitioner, when she made the
promise, had acted with the knowledge and under the authority of the insurance
company. But it is clear that the premium of P93,180.00 was actually paid to the
insurance company in consideration of the policy taken out. Private respondent's checks
were issued for the account of Manila Bankers Life Insurance Corporation and so
encashed by the insurance company as the payee. Private respondent thus cannot
"demand the return of what he has given" from petitioner because he did not, strictly
speaking, pay the amount of P93,180.00 to petitioner.

Kenneth B. Minglana

GREAT PACIFIC LIFE ASSURANCE vs. CA AND NGO HING


89 SCRA 543 / April 30, 1979

FACTS:

Ngo Hing applied for an insurance on the life of his child Helen Go with Great
Pacific Life for a 20 year endowment policy thru its agent Mr. Mondragon. He concealed
the fact that his daughter was a mongoliod child. After he paid to Mondragon the initial
premium, Mondragon issued a BINDING DEPOSIT RECEIPT at the back of which are
conditions among them is that “the company must be first satisfied that the applicant is
insurable on standard rates and if the company disapproves, the insurance shall never
be considered in force at anytime”. Mondragon then forwarded his application form to
the main office. Later on Great Pacific sent a letter disapproving the insurance
application saying that the 20 year endowment plan is not available for minors. Instead
of notifying Ngo Hing of the disapproval, Mondragon sent back a letter coupled with
strong recommendation that said application for a 20-year endowment plan be
accepted. In the course of the exchange of letters, Helen died.

ISSUE:

1. Is the binding receipt considered as a temporary contract of life insurance?


2. Was there concealment? If so, what is its effect?

RULING:

1. The binding receipt does not prove the existence of a temporary contract of life
insurance. The one issued to Ngo Hing lays down the conditions before such
receipt could bind Great Pacific one of which is that there must be acceptance of
the application first before it can create its binding effect. The receipt was nothing
more than an acknowledgement that the application and premium is received by
its branch. A contract of insurance, like other contracts, must be assented to by
both parties either in person or by their agents ... The contract, to be binding from
the date of the application, must have been a completed contract, one that
leaves nothing to be done, nothing to be completed, nothing to be passed upon,
or determined, before it shall take effect. There can be no contract of insurance
unless the minds of the parties have met in agreement.
2. There is concealment. He deliberately concealed that his daughter is a
mongoliod. Such medical condition is a fact material to the risk assumed by the
insurance company. The contract of insurance is one of utmost good faith.
Concealment, whether intentional or not entitles the insurer to rescind the
contract of insurance.
Sundae June A. Jugao

NG GAN ZEE vs. ASIAN CRUSADER LIFE ASSURANCE CORP.


122 SCRA 61

FACTS:

Ng Gan Zee was the beneficiary of a life insurance policy which is


procured by his husband Kwon Nam. Kwong Nam died of cancer of the liver
with metastasis. All premiums had been religiously paid at the time of his
death. Subsequently, Ng presented a claim to appellant for payment of the
face value of the policy and submitted the required proof of death of the
insured. Appellant denied the claim on the ground that the answers given by
the insured to the questions in his application for life insurance were untrue
and the insured was guilty of misrepresentation. It was found that prior to his
application, Kwong was diagnosed to have peptic ulcers, and that during the
operation what was removed from Kwong’s body was actually a portion of
the stomach and not tumor. Appellant further maintains that when the
insured was examined in connection with his application for life insurance, he
gave the appellant's medical examiner false and misleading information as
to his ailment and previous operation.

ISSUE:

Is there misrepresentation as to avoid the insurer from liability?

RULING:

No. It bears emphasis that Kwong Nam had informed the appellant's medical
examiner of the tumor. His statement that said tumor was "associated with ulcer of the
stomach" should be construed as an expression made in good faith of his belief as
to the nature of his ailment and operation. Kwong did not have sufficient
knowledge as to distinguish between a tumor and a peptic ulcer. His
statement therefore was made in good faith. Section 27 of the Insurance Law
provides, “Such party a contract of insurance must communicate to the
other, in good faith, all facts within his knowledge which are material to the
contract, and which the other has not the means of ascertaining, and as to
which he makes no warranty.” "Concealment exists where the assured had
knowledge of a fact material to the risk, and honesty, good faith, and fair
dealing requires that he should communicate it to the assurer, but he
designedly and intentionally withholds the same." While the information
communicated was imperfect, the same was sufficient to have induced
appellant to make further inquiries about the ailment and operation of the
insured. Section 32 of Insurance Law: Section 32. The right to information of
material facts may be waived either by the terms of insurance or by neglect
to make inquiries as to such facts where they are distinctly implied in other
facts of which information is communicated. Where a question appears to be
not answered at all or to be imperfectly answered, and the insurers issue a
policy without any further inquiry, they waive the imperfection of the answer
and render the omission to answer more fully immaterial. Asian’s failure to
inquire constituted a waiver of the imperfection in the answer.

Junyvil B. Tumbaga

NEW LIFE ENTERPRISES V. COURT OF APPEALS


G.R. No. 94071, 207 SCRA 669 March 31, 1992

FACTS:

Julian Sy and Jose Sy Bang have formed a business partnership in the City of
Lucena. Under the business name of New Life Enterprises, the partnership engaged in
the sale of construction materials at its place of business, a two storey building situated
at Iyam, Lucena City. Julian Sy insured the stocks in trade of New Life Enterprises with
Western Guaranty Corporation, Reliance Surety and Insurance. Co., Inc., and Equitable
Insurance Corporation. On May 15, 1981, Western Guaranty Corporation
issued Fire Insurance Policy to New Life Enterprises for P350,000 and it was
renewed on May, 13, 1982. On July 30,1981, Reliance Surety and Insurance
Co., Inc. issued Fire Insurance Policy to New Life Enterprises for P300,000 and
on November 12, 1981 for additional P700,000. On February 8, 1982,
Equitable Insurance Corporation issued Fire Insurance Policy to New Life
Enterprises for P200,000. Meanwhile on October 19, 1982, a fire electrical in
nature destroyed the stock in trade worth P1,550,000 of New Life
Enterprises . Julian Sy went to Reliance to claim but he was refused. Same
thing happened with the others who were sister companies. Accordingly, Sy
violated the "Other Insurance Clause". The RTC favored New Life and against
the three insurance companies. The CA reversed the ruling of the RTC for
failure to state or endorse the other insurance coverage.

ISSUE:

Whether or not Sy can claim against the three insurance companies for
violating the "Other Insurance Clause"

RULING:

No. The terms of the contract are clear and unambiguous. The insured is specifically
required to disclose to the insurer any other insurance and its particulars which he may
have effected on the same subject matter. The knowledge of such insurance by the
insurer’s agents, even assuming the acquisition thereof by the former, is not the “notice”
that would estop the insurers from denying the claim. Conclusion of the trial court that
Reliance and Equitable are “sister companies” is an unfounded conjecture drawn from
the mere fact that Yap Kam Chuan was an agent for both companies which also had the
same insurance claims adjuster. Availment of the services of the same agents and
adjusters by different companies is a common practice in the insurance business and
such facts do not warrant the speculative conclusion of the trial court. The conformity of
the insured to the terms of the policy is implied from his failure to express any
disagreement with what is provided for. A clear misrepresentation and a vital one
because where the insured had been asked to reveal but did not, that was deception –
guilty of clear fraud. The total absence of such notice nullifies the policy. Assuming
arguendo that petitioners felt the legitimate need to be clarified as to the policy condition
violated, there was considerable lapse of time from their receipt of the insurer’s
clarificatory letter dated March 302, 1983, up to the time the complaint was filed in court
on January 31, 1984. The one-year prescriptive period was yet to expire on November
29, 1983 or about eight (8) months from the receipt of the clarificatory letter, but
petitioners let the period lapse without bringing their action in court.

William Z. Radaza

SUNLIFE ASSURANCE COMPANY OF CANADA vs. CA


245 SCRA 268; June 22, 1995

FACTS:

Robert John Bacani procured a life insurance contract for himself from petitioner
Sunlife. The insurer waived medical examination of the insured. The insured died in a
plane crash, so her mother, who was the designated beneficiary, filed a claim with
petitioner Sunlife seeking the benefits of the insurance policy taken by her son. Sunlife
refused payment and returned the premium to insured’s mother on the ground that there
was concealment. Sunlife was able to show proof that the insured failed to disclose that
he had been diagnosed for renal failure prior to the application for insurance.

The beneficiary argued that the facts concealed by the insured were made in
good faith and under belief that they need not be disclosed, and that the health history
of the insured was immaterial since the insurance policy was non-medical. The CA ruled
that Sunlife cannot avoid its obligation by claiming concealment, because the cause of
death was unrelated to the facts concealed by the insured.

ISSUE:

Whether or not there was concealment which would avoid the insurance policy
and relieve insurer of its liability.
RULING:

There was concealment which avoided the insurance policy. Section 26 of the
Insurance Code is explicit in requiring a party to a contract of insurance to communicate
to the other, in good faith, all facts within his knowledge which are material to the
contract and as to which he makes no warranty, and which the other has no means of
ascertaining. Materiality is to be determined not by the event, but solely by the probable
and reasonable influence of the facts upon the party to whom communication is due, in
forming his estimate of the disadvantages of the proposed contract or in making his
inquiries.

The information which the insured failed to disclose were material and relevant to
the approval and issuance of the insurance policy. The matters concealed would have
definitely affected petitioner's action on his application, either by approving it with the
corresponding adjustment for a higher premium or rejecting the same. It is well settled
that the insured need not die of the disease he had failed to disclose to the insurer. It is
sufficient that his non-disclosure misled the insurer in forming his estimates of the risks
of the proposed insurance policy or in making inquiries. The waiver by the insurer of the
medical examination renders even more material the information required of the
applicant concerning previous condition of health because such information constitutes
an important factor to the insurer whether to issue the policy or not.

Aurora Luanne R. Cembrano

PAN MALAYAN UNSURANCE CORP. vs. CA


184 SCRA 54 / April 3, 1990

FACTS:

PANMALAY insured a car registered under the name of Canlubang Automotive


Resources Corporation (CANLUBANG). The insured car was hit and it suffered damages.
PANMALAY defrayed the cost of repair of the insured car under the “own damage” coverage of
the policy and was now seeking payment from the parties liable under its right of subrogation.
However, despite repeated demands, private respondents Erlinda Fabie and her unknown driver
refused to pay the claim. Private respondents refuted stating that PANMALAY had no cause of
action against them. Payment under the “own damage” clause of the insurance policy precluded
subrogation under Art. 2207 of the Civil Code since indemnification thereunder was made on the
assumption that there was no wrongdoer or no third party at fault.

ISSUE:

Whether or not PANMALAY may institute an action to recover the amount it had paid to
CANLUBANG against private respondents.

RULING:

PANMALAY may institute an action against private respondents. The SC held that when
PANMALAY used the phrase “own damage” – a phrase which is not found in the insurance
policy – to define the basis for its settlement of CANLUBANG’s claim, it simply meant that it
had assumed to reimburse the costs for repairing the damage to the insured vehicle.

Article 2207 of the Civil Code is founded on the well-settled principle of subrogation,
and payment by the insurer to the assured operates as an equitable assignment to the former of all
remedies which the latter may have against the third party whose negligence or wrongful act
caused the loss. The right of subrogation is not dependent upon, nor does it grow out of, any
privity of contract or upon written assignment of claim. It accrues simply upon payment of the
insurance claim by the insurer.

Christine Mae P. Navarra

PERLA COMPANIA DE SEGUROS, INC. vs. CA


185 SCRA 741 / May 28, 1990

FACTS:

Private respondent Milagros Cayas was the registered owner of a Mazda bus which was
sured with PerlaCompania de Seguros, Inc. (PCSI). The bus figured in an accident in Naic,
Cavite injuring several of its passengers. One of them, 19-year old Edgardo Perea, sued Milagros
Cayas for damages in the CFI of Cavite, Branch, while three others, agreed to a settlement of
P4,000.00 each. At the pre-trial, Milagros Cayas failed to appear and hence, she was declared as
in default. After trial, the court rendered a decision in favor of Perea of which Cayas was ordered
to compensate the latter with damages. Cayas filed a complaint with the CFI, seeking
reimbursement from PCSI for the amounts she paid to all victims, alleging that the latter refused
to make such reimbursement notwithstanding the fact that her claim was within its contractual
liability under the insurance policy.The decision of the CA affirmed in toto the decision of RTC
of Cavite, the dispostive portion of which states:

“IN VIEW OF THE FOREGOING, judgment is hereby rendered ordering


defendant PCSI to pay plaintiff Milagros Cayas the sum of P50,000.00 under its
maximum liability as provided for in the insurance policy; and the sum of
P5,000.00 as reasonable attorney's fee with costs against said defendant.”

In this petition for review on certiorari, petitioner seeks to limit its liability only to the
payment made by private respondent to Perea and only up to the amount of P12, 000.00. It
altogether denies liability for the payments made by Cayas to the other 3 injured passengers
totalling to P12, 000.00.
ISSUE:

How much should Petitioner PCSI pay?

RULING:

The decision of the CA is modified, petitioner is only to pay Cayas P12,000.00.The


insurance policy provides: “..no admission, offer, promise or payment shall be made by or on
behalf of the insured without the written consent of the Company…”It being specifically
required that petitioner's written consent be first secured before any payment in settlement of any
claim could be made, private respondent is precluded from seeking reimbursement of the
payments made the three other injured passengers in view of her failure to comply with the
condition contained in the insurance policy. Also, the insurance policy involved explicitly limits
petitioner's liability to P12,000.00 per person and to P50,000.00 per accident. Clearly, the
fundamental principle that contracts are respected as the law between the contracting parties
finds application in the present case. Thus, it was error on the part of the trial and appellate
courts to have disregarded the stipulations of the parties and to have substituted their own
interpretation of the insurance policy.The court also observed that although Milagros Cayas was
able to prove a total loss of only P44,000.00, petitioner was made liable for the amount of
P50,000.00, the maximum liability per accident stipulated in the policy. This is patent error. An
insurance indemnity, being merely an assistance or restitution insofar as can be fairly
ascertained, cannot be availed of by any accident victim or claimant as an instrument of
enrichment by reason of an accident.

Albert G. Cong

DELSAN TRANSPORT LINES vs. COURT OF APPEALS


369 SCRA 24 (2001)

FACTS:

Caltex Phil. (Caltex) entered into a contract of affreightment with Delsan


Transport Lines (Delsan) for a period of one year whereby Delsan agreed to transport
Caltex industrial fuel oil from Batangas refinery to different parts of the country. MT
Maysun, carrying Caltex’s goods, among others, set sail for Zamboanga City but sank
near Panay Gulf. The shipment was insured with American Home Assurance
Corporation (American). Subsequently, American paid Caltex the sum of
Php.5,096,635.57. Exercising its right of subrogation, American demanded from the
Delsan the same amount paid to Caltex but to no avail. Due to its failure to collect from
the Delsan, American filed a complaint with the RTC of Makati City but the trial court
dismissed the complaint, finding the vessel to be seaworthy and that the incident was
due to a force majeure, thus exempting the petitioner from liability. However, the
decision of the trial court was reversed by the CA, owing to the report of PAGASA that
the weather was normal and that it was impossible for the vessel to sink that day.

ISSUE:

Does payment made by United for the insured value of the lost cargo amount to
an admission that the vessel was seaworthy, thus precluding any action for recovery
against Delsan?

RULING:
No. The payment by the private respondent for the insured value of the lost cargo
operates as waiver of its right to enforce the term of the implied warranty against Caltex
under the marine insurance policy. However, the same cannot be validly interpreted as
an automatic admission of the vessel’s seaworthiness by the private respondent as to
foreclose recourse against the petitioner for any liability under its contractual obligation
as common carrier. The fact of payment grants the private respondent subrogatory right
which enables it to exercise legal remedies that otherwise be available to Caltex as
owner of the lost cargo against the petitioner common carrier.

Rogaciano A. Quico III

THE CAPITAL INSURANCE & SURETY CO., INC vs. PLASTIC ERA CO., INC.
65 SCRA 134 / July 18, 1975

FACTS:

Capital Insurance & Surety Co., Inc (Capital Insurance) delivered to Plastic Era
Manufacturing Co., Inc (Plastic Era) its Open Fire Policy No.22760 wherein the former
undertook to insure the latter’s building, equipments, raw materials, products and
accessories. The policy expressly provides that if the property insured would be
destroyed or damaged by fire after the payment of the premiums, at anytime between
the Dec. 15 1960 and one o'clock in the afternoon of the Dec. 15, 1961, the insurance
company shall make good all such loss or damage in an amount not exceeding P100k.
Plastic Era failed to pay its premium and instead executed an acknowledgment receipt
promising to pay 30 days after date.

On Jan.8,1961, Plastic Era delivered a check as partial payment of the insurance


premium worth 1k. However, it was dishonored by the Bank of America for lack of funds
on Feb.20, 1961. (Note: premium due date was on Jan.16,1961)

Unexpectedly, the property insured by Plastic Era was destroyed by fire on


Jan.18, 1961 (that’s 2 days after the premium became due). Plastic Era filed a claim for
indemnity.

ISSUE:
Whether or not a contract of insurance has been duly perfected between the
petitioner, Capital Insurance, and respondent Plastic Era.

RULING:

Yes, it has been perfected. In clear and unequivocal terms the insurance policy
provides that it is only upon payment of the premiums by Plastic Era that Capital
Insurance agrees to insure the properties of the former against loss or damage in an
amount not exceeding P100,000.00. Significantly, Capital Insurance accepted the
promise of Plastic Era to pay the insurance premium within thirty (30) days from the
effective date of policy. By so doing, it has implicitly agreed to modify the tenor of the
insurance policy and in effect, waived the provision therein that it would only pay for the
loss or damage in case the same occurs after the payment of the premium. Considering
that the insurance policy is silent as to the mode of payment, Capital Insurance is
deemed to have accepted the promissory note in payment of the premium.

The fact that the check issued by Plastic Era in partial payment of the promissory
note was later on dishonored did not in any way operate as a forfeiture of its rights
under the policy, there being no express stipulation therein to that effect.

Mary Christine Anthonette M. Salise-Punzalan

GREAT PACIFIC LIFE INSURANCE CORPORATION vs. CA


G.R. No. L-57308 / April 23, 1990

FACTS:

Private respondent Teodoro Cortez, upon the solicitation of an underwriter for the
petitioner Great Pacific Insurance Corporation, applied for a 20-year endowment policy for
P30,000. His application, with the requisite medical examination, was accepted and approved by
the company and in due course, Endowment Policy No. 221944 was issued in his name. It was
released for delivery on January 24, 1973, and was actually delivered to him by the underwriter,
Mrs. Siega on January 25, 1973. The effective date indicated on the face of the policy in question
was December 25, 1972. The annual premium was P1,416.60. Mrs. Siega assured him that the
first premium may be paid within the grace period of thirty (30) days from date of delivery of the
policy. In a letter dated June 1, 1973 (Exh. E), defendant advised plaintiff that Policy No. 221944
(Exh. A) was not in force. To make it enforceable and operative, plaintiff was asked to remit the
balance of P1,015.60 to complete his initial annual premium due December 15, 1972, and to see
Dr. Felipe V. Remollo for another full medical examination at his own expense.

Cortez' reaction to the company's act was to immediately inform it that he was cancelling
the policy and he demanded the return of his premium plus damages.
When the company ignored his demand, Cortez filed on August 14, 1973, a complaint for
damages in the Court of First Instance of Negros Oriental, docketed as Civil Case No. 5709,
entitled "Teodoro Cortez vs. Pacific Life Assurance Corporation.

ISSUE:

Whether Cortez is entitled to a refund of his premium

RULING:

Yes. When the petitioner advised private respondent on June 1, 1973, four months after
he had paid the first premium, that his policy had never been in force, and that he must pay
another premium and undergo another medical examination to make the policy effective, the
petitioner committed a serious breach of the contract of insurance. Petitioner should have
informed Cortez of the deadline for paying the first premium before or at least upon delivery of
the policy to him, so he could have complied with what was needful and would not have been
misled into believing that his life and his family were protected by the policy, when actually they
were not. And, if the premium paid by Cortez was unacceptable for being late, it was the
company's duty to return it. By accepting his premiums without giving him the corresponding
protection, the company acted in bad faith. Since his policy was in fact inoperative or ineffectual
from the beginning, the company was never at risk, hence, it is not entitled to keep the premium.

Ropa Arienza- Arpilleda

VALENZUELA VS. CA
191 SCRA 1 Oct. 19,1990

FACTS:

Petitioner Arturo P. Valenzuela is a General Agent of private respondent Philippine


American General Insurance Company, Inc. (Philamgen) since 1965 authorized to solicit and sell
in behalf of Philamgen all kinds of non-life insurance, and in consideration of services rendered
was entitled to receive the full agent's commission of 32.5% from Philamgen. Valenzuela
solicited marine insurance from one of his clients, the Delta Motors, Inc. from which he was
entitled to a commission of 32%. However, Valenzuela did not receive his full commission which
amounted to P1.6 Million from the P4.4 Million insurance coverage of the Delta Motors. During
the period 1976 to 1978, premium payments amounting to P1,946,886.00 were paid directly to
Philamgen and Valenzuela's commission to which he is entitled amounted to P632,737.00.

In 1977, Philamgen started to become interested in and expressed its intent to share in the
commission due Valenzuela on a fifty-fifty basis. Philamgen insisted on the sharing of the
commission with Valenzuela to which the latter firmly reiterated his objection to the proposals of
respondents. The pressures and demands, however, continued until the agency agreement itself
was finally terminated. Worse, despite the termination of the agency, Philamgen continued to
hold Valenzuela jointly and severally liable with the insured for unpaid premiums. The
petitioners sought relief by filing the complaint against the private respondents in the court a
quo.

ISSUE:

Whether or not the petitioners are liable to Philamgen for the unpaid and uncollected
premiums of the insured which the respondent court ordered Valenzuela to pay the latter.

RULING:

SC ruled that the respondent court erred in holding Valenzuela liable. There is no factual
and legal basis for the award. Under Section 77 of the Insurance Code, the remedy for the non-
payment of premiums is to put an end to and render the insurance policy not binding — Sec.
77 ... Notwithstanding any agreement to the contrary, no policy or contract of insurance is valid
and binding unless and until the premiums thereof have been paid except in the case of a life or
industrial life policy whenever the grace period provision applies (P.D. 612, as amended
otherwise known as the Insurance Code of 1974)

In addition, the SC cited the ruling made in Philippine Phoenix Surety and Insurance,
Inc. v. Woodworks, Inc. that the non-payment of premium does not merely suspend but puts an
end to an insurance contract since the time of the payment is peculiarly of the essence of the
contract. Moreover, an insurer cannot treat a contract as valid for the purpose of collecting
premiums and invalid for the purpose of indemnity.

The foregoing findings are buttressed by Section 776 of the Insurance Code which now
provides that no contract of Insurance by an insurance company is valid and binding unless and
until the premium thereof has been paid, notwithstanding any agreement to the contrary.

Perforce, since admittedly the premiums have not been paid, the policies issued have
lapsed. The insurance coverage did not go into effect or did not continue and the obligation of
Philamgen as insurer ceased. Hence, for Philamgen which had no more liability under the lapsed
and inexistent policies to demand, much less sue Valenzuela for the unpaid premiums would be
the height of injustice and unfair dealing. In this instance, with the lapsing of the policies through
the nonpayment of premiums by the insured there were no more insurance contracts to speak of.
As the Court held in the Philippine Phoenix Surety case "the non-payment of premiums does not
merely suspend but puts an end to an insurance contract since the time of the payment is
peculiarly of the essence of the contract."
Silhouette B. Adobas

PHIL. PRYCE ASSURANCE CORP vs CA


230 SCRA 164/ 21 Feb 1994
FACTS:

Petitioner, Interworld Assurance Corporation (the company now carries the


corporate name Philippine Pryce Assurance Corporation), was the butt of the complaint
for collection of sum of money, filed on May 13, 1988 by respondent, Gegroco, Inc.
before the Makati Regional Trial Court, Branch 138. The complaint alleged that
petitioner issued two surety bonds (No. 0029, dated July 24, 1987 and No. 0037, dated
October 7, 1987) in behalf of its principal Sagum General Merchandise for FIVE
HUNDRED THOUSAND (P500,000.00) PESOS and ONE MILLION (1,000,000.00)
PESOS, respectively.

Petitioner hinges its defense on two arguments, namely: a) that the checks
issued by its principal which were supposed to pay for the premiums, bounced, hence
there is no contract of surety to speak of; and 2) that as early as 1986 and covering the
time of the Surety Bond, Interworld Assurance Company (now Phil. Pryce) was not yet
authorized by the insurance Commission to issue such bonds.

ISSUE:
Is the contract of surety valid?

RULING:

Yes, the contract of surety is valid. The Insurance Code states that:

Sec. 177. The surety is entitled to payment of the premium as soon as the contract
of suretyship or bond is perfected and delivered to the obligor. No contract of
suretyship or bonding shall be valid and binding unless and until the premium
therefor has been paid, except where the obligee has accepted the bond, in which
case the bond becomes valid and enforceable irrespective of whether or not the
premium has been paid by the obligor to the surety. . . .

The petitioner, in its answer, admitted to have issued the bonds subject matter of the
original action.
On the other hand, petitioner's defense that it did not have authority to issue a Surety
Bond when it did is an admission of fraud committed against respondent. No person can claim
benefit from the wrong he himself committed. A representation made is rendered conclusive
upon the person making it and cannot be denied or disproved as against the person relying
thereon.
Kenneth B. Minglana

AMERICAN HOME ASSURANCE vs. ANTONIO CHUA


309 SCRA 250 / June 28, 1999

FACTS:

Antonio Chua sought renewal of a fire insurance policy issued by American


Home. As payment for the renewal, he issued a check to an agent of American Home,
James Uy. The latter then issued a renewal certificate in favor of Antonio Chua. A day
after he issued the check, the subject matter of the insurance –a building owned by
Chua, was totally grazed by fire. Chua filed to claim the proceeds. American home
denied the claim contending that no contract of insurance existed at the time of loss
since Chua did not pay the premium. It cited Section 77 which states that unless and
until a premium is paid there is no insurance. The check issued did not operate as
payment since under the Civil Code, a check can only effect payment the day it is
encashed. Considering that the check was encashed approximately 4 days after its
deposit, the policy in question remains expired at the time of loss. Moreover, American
Home contends that the non-disclosure of Chua that the same building was insured
against fire by other insurers render the policy void.

ISSUES:

1. WON a valid contract of Insurance exists at the time of loss


2. What is the effect of non-disclosure of Chua of the other insurance taken on the
same building?

RULING:

1. A valid contract of insurance exists at the time of loss. A general rule in insurance
laws is that unless the premium has been paid, the insurance policy is not valid
and binding except in life and industrial life insurance. In the instant case, it is
not disputed that the check issued by Chua was honored when presented and
petitioner forthwith issued a certificate of renewal categorically stating that the
premium has been paid. Section 78 of the ICP provides that “ An
acknowledgment in a policy or contract of insurance of the receipt of premium is
conclusive evidence of its payment, so far as to make the policy binding,
notwithstanding any stipulation therein that it shall not be binding until the
premium is actually paid.”

2. Normally, non-disclosure of other insurance on the same subject matter would


avoid the policy however, in the instant case American Home is estopped from
raising the said argument since it is its own loss adjuster which admitted that he
knows full well prior to the loss that the same subject matter has been insured by
different insurance companies. Prior knowledge of other insurances defeats the
argument that the insurer is deceived into entering the contract of insurance.
Sundae June A. Jugao

UCPB GENERAL INSURANCE v. MASAGANA TELEMART


356 SCRA 307

FACTS:

Masagana Telamart’s obtained from UCPB five (5) insurance policies on its
various properties for the period from 22 May 1991 to 22 May 1992.
On March 1992 or 2 months before policy expiration, UCPB evaluated the
policies and decided not to renew them upon expiration of their terms on 22
May 1992. UCPB advised Masagana’s broker of its intention not to renew the
policies. On April 1992, UCPB gave written notice to Masagana of the non-
renewal of the policies. On June 1992 [policy already expired], Masagana’s
property covered by 3 UCPB-issued policies was razed by fire. On 13 July
1992, Masagana presented to UCPB’s cashier 5 manager's checks,
representing premium for the renewal of the policies for another year. It was
only on the following day, 14 July 1992, when Masagana filed with UCPB
a formal claim for indemnification of the insured property razed by fire. On
the same day, UCPB returned the 5 manager's checks, and rejected
Masagana’s claim since the policies had expired and were not renewed, and
the fire occurred on 13 June 1992 (or before tender of premium payment).
Masagana filed a civil complaint for recovery of the face value of the policies
covering the insured property razed by fire. RTC ruled in favor of Masagana,
which is also affirmed by CA, holding that following previous
practice, Masagana was allowed a 60-90 day credit term for the renewal of
its policies, and that the acceptance of the late premium payment suggested
that payment could be made later.
ISSUE:

Whether or not the fire insurance policies had expired and Masagana’s
claims should be rejected.
RULING:

The fire insurance policies had expired and Masagana’s claims should
be rejected. An insurance policy, other than life is not valid and binding until
actual payment of the premium. Any agreement to the contrary is void. The
parties may not agree expressly or impliedly on the extension of credit or
time to pay the premium and consider the policy binding before actual
payment. In the present case, the payment of the premium for renewal of
the policies was tendered a month after the fire occurred. Masagana did not
even give UCPB a notice of loss within a reasonable time after occurrence of
the fire.
Junyvil B. Tumbaga

VDA. DE MAGLANA vs. CONSOLACION


[GR 60506, 6 August 1992], 212 SCRA 268

FACTS:

Lope Maglana was an employee of the Bureau of Customs whose work station
was at Lasa, in Davao City. On 20 December 1978, early morning, Lope Maglana was
on his way to his work station, driving a motorcycle owned by the Bureau of Customs. At
Km. 7, Lanang, he met an accident that resulted in his death. The PUJ jeep that
bumped the deceased was driven by Pepito Into, operated and owned by Destrajo.
Consequently, the heirs of Lope Maglana, Sr., filed an action for damages and
attorney's fees against operator Patricio Destrajo and the Afisco Insurance Corporation
(AFISCO) before the then Court of First Instance . An information for homicide thru
reckless imprudence was also filed against Pepito Into. During the pendency of the civil
case, Into was sentenced and found guilty in the criminal case t, and to indemnify the
heirs of Lope Maglana, Sr. plus moral and exemplary damages. On the other hand, the
lower court rendered a decision finding that Destrajo had not exercised sufficient
diligence as the operator of the jeepney. The court ordered Destrajo to pay the heirs of
Maglana for loss of income; the sum of P12,000.00 which amount shall be deducted in
the event judgment against the driver, accused Into. The court ordered the insurance
company to reimburse Destrajo whatever amounts the latter shall have paid only up to
the extent of its insurance coverage. The heirs of Maglana contended that AFISCO
should not merely be held secondarily liable because the Insurance Code provides that
the insurer's liability is "direct and primary and/or jointly and severally with the operator
of the vehicle, although only up to the extent of the insurance coverage."
.
ISSUE:

Whether AFISCO is solidarily liable with Destrajo.

RULING:

No. In Malayan Insurance Co., Inc. v. Court of Appeals, the Court had the
opportunity to resolve the issue as to the nature of the liability of the insurer and the
insured vis-a-vis the third party injured in an accident, where it ruled that "While it is true
that where the insurance contract provides for indemnity against liability to third
persons, such third persons can directly sue the insurer, however, the direct liability of
the insurer under indemnity contracts against third party liability does not mean that the
insurer can be held solidarily liable with the insured and/or the other parties found at
fault. The liability of the insurer is based on contract; that of the insured is based on
tort." The Court then proceeded to distinguish the extent of the liability and manner of
enforcing the same in ordinary contracts from that of insurance contracts. While in
solidary obligations, the creditor may enforce the entire obligation against one of the
solidary debtors, in an insurance contract, the insurer undertakes for a consideration to
indemnify the insured against loss, damage or liability arising from an unknown or
contingent event." The liability of AFISCO based on the insurance contract is direct, but
not solidary with that of Destrajo which is based on Article 2180 of the Civil Code. As
such, the heirs have the option either to claim the P15,000 from AFISCO and the
balance from Destrajo or enforce the entire judgment from Destrajo subject to
reimbursement from AFISCO to the extent of the insurance coverage.

William Z. Radaza

GSIS vs. CA
308 SCRA 559; June 21, 1999

FACTS:

The National Food Authority (NFA) was the owner of a Chevrolet truck
which was insured against liabilities for death of and injuries to third persons.
GSIS was the insurer. The said truck driven by Guillermo Corbeta collided
with a Toyota Tamaraw public utility vehicle. Five passengers of the Toyota
vehicle died while ten others sustained bodily injuries. An action for damages
was filed against NFA and the truck driver based on quasi-delict, and against GSIS as
insurer of the truck, and MIGC the insurer of the Toyota passenger vehicle. The trial
court awarded damages to the victims and injured parties, and held NFA, GSIS, and
MIGC jointly and severally (or solidarily) liable for the payment thereof. GSIS denies
solidary liability with NFA arguing that GSIS and NFA are liable under different
obligations. It asserts that the NFA’s liability is based on quasi-delict, while
GSIS’s liability is based on the contract of insurance.

ISSUE:

Whether or not GSIS, as insurer, is solidarily liable with NFA for the damage
caused to third persons as a result of the injuries arising from vehicular accidents
caused by the insured truck.

RULING:

GSIS cannot be made solidarily liable for all the damage caused by the
negligence of the driver of NFA. Nevertheless, the victims may proceed directly against
GSIS as insurer, but it will be liable only up to the extent of the amount of the insurance
policy. It is now established that the injured or the heirs of a deceased victim
of a vehicular accident may sue directly the insurer of the vehicle.

Common carriers are required to secure Compulsory Motor Vehicle


Liability Insurance coverage as provided under Sec. 374 of the Insurance
Code, precisely for the benefit of victims of vehicular accidents and to extend
them immediate relief. The purpose of statutes enabling an injured person to
proceed directly against the insurer is to protect them against the insolvency
of the insured who causes such injury, and to give such injured person a
certain beneficial interest in the proceeds of the policy.
While it is true that where the insurance contract provides for indemnity
against liability to third persons, and such third persons can directly sue the
insurer, the direct liability of the insurer under indemnity contracts against
third party liability is only up to the extent of what was provided for by the
contract of insurance. As therein provided, the maximum indemnity for death
was twelve thousand (P12,000.00) pesos per victim.

Aurora Luanne R. Cembrano

TIU vs. ARRIESGADO


437 SCRA 426 / Sept. 1, 2004

FACTS:

A passenger bus owned by Petitioner Tiu collided with a cargo truck


which was parked along the right side of the national highway with damaged
rear tires. The collision injured Private Respondent Pedro Arriesgado and
killed his wife.

Arriesgado then filed a complaint for breach of contract of carriage,


damages and attorney’s fees against Tiu. In turn, Tiu filed a Third-Party
Complaint against respondent Philipine Phonenix Surety and Insurance, Inc.
PPSII, stating that the passenger bus is covered by a common carrier liability
insurance issued by PPSII. Tiu insists that PPSII is liable to him for
contribution, indemnification and/or reimbursement.

PPSII, for its part, admitted that while it had attended to and settled
the claims of the other injured passengers, respondent Arriesgado’s claim
remained unsettled as it was beyond the limits of liability as so stated in the
contract. The insurance contract expressly provided therein that the limit of
the insurer’s liability for each person was P12,000, while the limit per
accident was pegged at P50,000.

Petitioner Tiu contests that respondent PPSII should have settled the
said claim instead of just denying the same.

ISSUE:

What is the liability of PPSII as insurer?

RULING:
As found by the Supreme Court, the insurance contract was issued
pursuant to the Compulsory Motor Vehicle Liability Insurance Law. It was
expressly provided therein that the limit of the insurer’s liability for each
person was P12,000, while the limit per accident was pegged at P50,000.

An insurer in an indemnity contract for third party liability is directly


liable to the injured party up to the extent specified in the agreement but it
cannot be held solidarily liable beyond that amount. The respondent PPSII
could not then just deny petitioner Tiu’s claim; it should have
paid respondent Arrisgado’s hospitalization expenses of P1,113.80 and
P12,000 for the death of his wife. The total amount of the claims, even when
added to that of the other injured passengers which PPSII claimed to have
settled, would not exceed the P50,000 limit under the insurance agreement.

Christine Mae P. Navarra

IVOR ROBERT DAYTON GIBSON vs. HON. PEDRO A. REVILLA, in his official
capacity as Presiding Judge of Branch XIII, Court of First Instance of Rizal, and
LEPANTO CONSOLIDATED MINING COMPANY
29 SCRA 219 / July 30, 1979

FACTS:

Lepanto Consolidated Mining Company (Lepanto) filed in the CFI of Rizal a complaint
with a plea for preliminary mandatory injunction against Malayan Insurance Company, Inc. The
civil suit thus instituted by Lepanto against Malayan was founded on the fact that on Sept. 9,
1971, Malayan issued Marine Open Policy No. LIDC-MOP-001/71 covering an shipments of
copper, gold and silver concentrates in bulk from Poro, San Fernando, La Union to Tacoma,
Washington or to other places in the United States. Thereafter, Malayan obtained reinsurance
abroad through Sedgwick, Collins & Co., Limited, a London insurance brokerage. The
Memorandum of Insurance issued by Sedgwick to Malayan on September 24, 1971 listed three
groups of underwriters or re-insurers and their reinsurance interest are as follows: Lloyds =
62.808%, Companies (I.L.U.) = 34.705%, and Other Companies = 2.487%.

At the top of the list of underwriting members of Lloyds is Syndicate No. 448, assuming
2.48% of the risk assumed by the reinsurer, which petitioner Ivor Robert Dayton Gibson claims
to be himself. Petitioner then filed a motion to intervene as defendant, which motion was denied
by the lower court.

ISSUE:
Whether the lower court committed reversible error in refusing the intervention of
petitioner Gibson in the suit between Lepanto and Malayan

RULING:

The respondent judge committed no error of law in denying petitioner’s motion to


intervene and neither has he abused his discretion in his denial of petitioner’s motion for
intervention. The Court agrees with the holding of the respondent Court that since movant
Gibson appears to be only one of several re-insurers of the risks and liabilities assumed by
Malayan Insurance Company, Inc., it is highly probable that other re- insurers may likewise
intervene. If petitioner is allowed to intervene, there is good and sufficient basis for the Court a
quoto declare that the trial between Lepanto and Malayan would be definitely disrupted and
would certainly unduly delay the proceedings between the parties especially at the stage where
Lepanto had already rested its case and that the issues would also be compounded as more
parties and more matters will have to be litigated.

The Court also holds that respondent Judge committed no reversible error in further
sustaining the fourth ground of Lepanto's Opposition to the Motion to Intervene that the rights, if
any, of petitioner are not prejudiced by the present suit and win be fully protected in a separate
action against him and his co-insurers by Malayan.Petitioner's contention that he has to pay once
Malayan is finally adjudged to pay Lepanto because of the very nature of a contract of
reinsurance and considering that the re-insurer is obliged 'to pay as may be paid thereon'
(referring to the original policies), although this is subject to other stipulations and conditions of
the reinsurance contract, is without merit. The general rule in the law of reinsurance is that the
re-insurer is entitled to avail itself of every defense which the re-insured (which is Malayan)
might urge in an action by the person originally insured (which is Lepanto) as provided in Sec.
1238 of the insurance code.
Albert G. Cong

AVON INSURANCE PLC vs. COURT OF APPEALS


278 SCRA 312 (1997)

FACTS:

Yupangco Cotton Mills (Yupangco) engaged to secure several of its properties


totaling P200,000,000.00 with Worldwide Security and Insurance Co. (Worldwide), the
contracts of which were covered by reinsurance treaties between Worldwide and
several foreign reinsurance companies, Avon Insurance PLC, et. al. (Avon, et. al.),
through CJ Boatrwright, acting as agent of Worldwide Surety and Insurance. Fire then
razed Yupangco’s properties on two separate occasions covered within the effectivity
period of the policies.

Worldwide and some of the foreign reinsurance companies made partial payment
to Yupangco. Worldwide then executed a Deed of Assignment to Yupangco,
acknowledging that a remaining balance of P19,444,447.75 was still due and assigned
to Yupangco all reinsurance proceeds still collectible from all the foreign reinsurance
companies.

Yupangco then filed a collection suit against Avon, et. al, the foreign reinsurers.
The service of summons was made through the office of the Insurance Commissioner.
In a Petition for Certiorari filed with the Court of Appeals, Avon, et. al.
submitted that the Philippine courts have no jurisdiction over them, being all
foreign corporations not doing business in the Philippines with no office,
place of business or agents in the Philippines. Yupangco contended that since the
reinsurers question the jurisdiction of the court they are deemed to have submitted to
the jurisdiction of the court.

ISSUE:

Do the Philippine courts have jurisdiction over international reinsurers who are
not doing business in the Philippines?

RULING:

No. The international reinsurers are not doing business in the Philippines and the
Philippine courts have not acquired jurisdiction over them. The reinsurance treaties
between the Avon, et. al. and Worldwide were made through an international insurance
broker, CJ Boatrwright, and not through any entity or means remotely connected with
the Philippines. Moreover, in Moris & Co. vs. Scandinavia Ins. Co., 279 U.S. 405
(1929), it was held that a reinsurance company is not doing business in a
certain state merely because the property or lives which are insured by the
original insurer company are located in that state. The reason for this is that
a contract of reinsurance is generally a separate and distinct arrangement
from the original contract of insurance, whose contracted risk is insured in
the reinsurance agreement. Hence, the original insured has generally no
interest in the contract of reinsurance.

Rogaciano A. Quico III

SUMMIT GUARANTY AND INSURANCE CO., INC. VS. DE GUZMAN


151 SCRA 389 / June 30, 1987

FACTS:

This petition for certiorari stems from three consolidated complaints filed against
petitioner, the facts of the three cases are as follows:

Jose Ledesma was the owner of a tractor which was bumped by a minibus
insured with petitioner for Third Party Liability. Ledesma immediately made a notice of
claim. Petitioner company advised private respondent to have car repaired by G.A.
Machineries, which was later estimated at an amount of Php21,000 and made
assurance of payment. Upon repair, respondent made several demands on petitioner
company because of repair shops warning that failure to pay would result in the
auctioning of the tractor to pay expenses. Petitioner company continued giving
assurance and promises to pay. Eventually, private respondent filed a formal complaint
with the Insurance Commission, which petitioner company moved to dismiss on ground
of prescription.
Geronima Pulmano was the owner of a jeep insured with petitioner company in
the amount of Php20,000. The jeep got into a vehicular accident which resulted in the
death of one of the victims and private respondent immediately filed a notice of accident
and claim. Petitioner company took no steps to process the claim so private
respondents brought their claim to the Insurance Commission, but petitioner company
still failed to settle. A complaint was eventually filed with the Court of First Instance of
Tarlac which petitioner company moved to dismiss on the ground of prescription.

Amelia Generao owned a passenger jeepney insured with petitioner under a


Vehicle Comprehensive Policy. The jeepney struck the van of a certain Mr. Hahn and
two days later Generao notified petitioner company and demanded payment on both
vehicles. Generao and petitioner company even had a dialogue at the office of
insurance company to settle the claim. Nonetheless, time passed without petitioner
company taking any final action. Mr. Hahn filed a complaint for damages against
Generao who, in response, filed a third party complaint against petitioner company
which in turn filed a motion to dismiss on the ground of prescription.

ISSUE:

Did the causes of action of private respondents already prescribe?

RULING:

NO. Petitioner company argues that under Section 384 of the Insurance Code,
even if the notice of claim was timely filed with the insurance company within the six
month period, if the action or suit that follows is filed beyond the one year period it
should necessarily be dismissed on the ground of prescription.

The Supreme Court finds absolutely nothing in the law which mandates that the
two periods must always concur. On the contrary, it is very clear that the one year
period is only required “in proper cases”. It is very obvious that petitioner company is
trying to use Section 384 of as a cloak to hide itself from its liabilities. In violation of its
duties to adopt and implement reasonable standards for the prompt investigation of
claims and to effectuate prompt, fair and equitable settlement of claims, and with
manifest bad faith, petitioner company devised means and ways of stalling the
settlement proceedings.

The one year period should be counted from the date of rejection by the insurer
as this is the time the cause of action accrues. Since in these cases there has yet been
no accrual of cause of action, prescription has not yet set in.

NOTE: Section 384 has been amended as follows, “…Action or suit for recovery
of damage due to loss or injury must be brought in proper cases, with the Commissioner
or the Courts within one year from denial of the claim, otherwise the claimants right of
action shall prescribe.”

Topic: Article 1263 - In an obligation to deliver a generic thing, the loss or


destruction of anything of the same kind does not extinguish the obligation
Mary Christine Anthonette M. Salise-Punzalan
COUNTRY BANKERS INSURANCE CORP. (Formerly Country Bankers Insurance &
Surety Co. Inc.) vs. THE TRAVELLERS INSURANCE AND SURETY CORP., and THE
HONORABLE COURT OF APPEALS
G.R. No. 82509 August 16, 1989

FACTS:

The Toyota Land Cruiser insured to petitioner was bumped by Isuzu Cargo Truck.Having
suffered, the Toyota Land Cruiser, its owner, declared a total loss and claimed the proceeds of the
insurance policy issued by petitioner Country Bankers Insurance Corporation. As subrogee to all
rights and causes of action of PTCI, petitioner demanded reimbursement from the driver and
owner of the Isuzu Cargo truck and from private respondent travellers Insurance but the latter
failed to act on petitioner's claim forcing the petitioner to file a complaint before the RTC
rendered a decision in favor of the petitioner and ordered private respondent to pay petitioner the
amount paid to PTCI, but dismissed the complaint as against the other two defendants.

On appeal, the Court of Appeals (CA) affirmed the finding of the RTC and private
respondent is liable to herein petitioner as the subrogee to all the rights and causes of action of
the owner of the damaged Toyota Land Cruiser. Nevertheless, the CA dismissed the complaint on
the ground that petitioner's cause of action had prescribed.

Defendant's defense that the action has prescribed is found meritorious. The accident
occurred on 24 May 1979, but the complaint was not filed until 14 October 1980, or almost
seventeen (17) months after the accident. Section 384 of the Insurance Code mandates that the
"(a)ction or suit for recovery of damage due to loss or injury must be brought, in proper cases,
with the courts within one year from the date of the accident, otherwise the claimant's right of
action shall prescribe.". . .

ISSUE:

Issue of whether the one-year prescriptive period under Section 384 of the Insurance
Code, prior to its amendment by Batas PambansaBlg. 874, should commence to run from the
date of the accident or from the rejection of the claim by the insurer.

RULING:

No. To prevent the insurance company from evading its responsibility to the
insured through this clever scheme, and to protect the insuring public against similar
acts by other insurance companies, the Court held that the one-year period under
Section 384 should be counted not from the date of the accident but from the date of the
rejection of the claim by the insurer [Summit, supra, at 397]. The Court further held that
it is only from the rejection of the claim by the insurer that the insured's cause of action
accrued since a cause of action does not accrue until the party obligated refuse,
expressly or impliedly, to comply with its duty [ACCFA v. Alpha Insurance and Surety
Co., G.R. No. L-24566, July 29,1968, 24 SCRA 151].
Ropa Arienza- Arpilleda

TRAVELLERS INSURANCE & SURETY CORPORATION VS. COURT OF APPEALS


272 SCRA 536 May 22, 1997

FACTS:

The complainant lumped the erring taxicab driver, the owner of the taxicab, and
the alleged insurer of the vehicle which featured in the vehicular accident (leading to the
death of his mother) into one complaint. The erring taxicab was allegedly covered by a
third-party liability insurance policy issued by petitioner Travellers Insurance & Surety
Corporation.

Private respondent filed a complaint for damages against Armando Abellon as


the owner of the Lady Love Taxi and Rodrigo Dumlao as the driver of the Lady Love
taxicab that bumped private respondent’s mother. Subsequently, private respondent
amended his complaint to include petitioner as the compulsory insurer of the said
taxicab under Certificate of Cover No. 1447785-3.

After trial, the trial court rendered judgment in favor of private respondent and
subsequently affirmed by the respondent CA.

Petitioner appealed the decision of the respondent Court of Appeals mainly


contending that it did not issue an insurance policy as compulsory insurer of the Lady
Love Taxi and that, assuming arguendo that it had indeed covered said taxicab for third-
party liability insurance, private respondent failed to file a written notice of claim with
petitioner as required by Section 384 of P.D. No. 612, otherwise known as the Insurance
Code.

ISSUE:

Whether or not private respondent has a right to implead and sue the petitioner-
insurer as party defendant to the case.
RULING:

The right of the person injured to sue the insurer of the party at fault (insured),
depends on whether the contract of insurance is intended to benefit third persons also
or on the insured. Under the contract of insurance, a policy whereby the insurer agreed
to indemnify the insured against all sums which the Insured shall become legally liable
to pay in respect of a death of or bodily injury to any person is one for indemnity against
liability; from the fact then that the insured is liable to the third person, such third person
is entitled to sue the insurer. And the test applied has been this: Where the contract
provides for indemnity against liability to third persons, then third persons to whom the
insured is liable can sue the insurer. Where the contract is for indemnity against actual
loss or payment, then third persons cannot proceed against the insurer, the contract
being solely to reimburse the insured for liability actually discharged by him thru
payment to third persons, said third persons’ recourse being thus limited to the insured
alone.

Apparently, the trial court did not distinguish between the private respondent’s
cause of action against the owner and the driver of the Lady Love taxicab and his cause
of action against petitioner. While it is true that where the insurance contract provides
for indemnity against liability to third persons, such third persons can directly sue the
insurer, however, the direct liability of the insurer under indemnity contracts against
third-party liability does not mean that the insurer can be held solidarily liable with the
insured and/or the other parties found at fault. The liability of the insurer is based on
contract; that of the insured is based on tort.

When petitioner asseverates, thus, that no written claim was filed by private
respondent and rejected by petitioner, and private respondent does not dispute such
asseveration through a denial in his pleadings, we are constrained to rule that
respondent appellate court committed reversible error in finding petitioner liable under
an insurance contract the existence of which had not at all been proven in court. Even
if there were such a contract, private respondent’s cause of action can not prevail
because he failed to file the written claim mandated by Section 384 of the Insurance
Code. He is deemed, under this legal provision, to have waived his rights as against
petitioner-insurer.

Thus, the complaint against Travellers Insurance & Surety Corporation in said
case was ordered dismissed.
Silhouette B. Adobas

PHIL. HOME ASSURANCE CORP v COURT OF APPEALS


301 SCRA 423/ 21 Jan 1999

FACTS:

Petitioners are the Philippine Home Assurance Corporation (PHAC), the Philippine
American Accident Insurance Company (PAAIC), the Philippine American General Insurance
Company (PAGIC), and the American International Underwriters (Phils.), Inc. (AIUPI), which
are domestic corporations engaged in the insurance business.

From January to June 1986, they paid under protest the total amount of P10,456,067.83
as documentary stamp taxes on various life and non-life insurance policies issued by them,
broken down as follows:

PHAC 1,714,459.00
PAAIC 68,046.00
PAGIC 3,816,973.00
AIUPI 4,856,589.83
TOTAL P10,456,067.83

On August 4, 1987, petitioners filed separate claims for refund from the Bureau of
Internal Revenue. They alleged that the premiums on the insurance policies issued by them had
not been paid thus, in accordance with 77 of the Insurance Code, no documentary stamp taxes
were due on the policies.

ISSUE:

Are life and non-life insurance policies subject to documentary stamp taxes
pursuant to 183 and 184 of the National Internal Revenue Code by their mere
issuance?

RULING:

Yes, they are. The Court of Tax Appeal correctly characterized a documentary
stamp tax as in the nature of an excise tax. As such, it is imposed on the privilege of
conducting a particular business or transaction and not on the business or transaction
itself. Thus, the documentary stamp tax on insurance policies is, in effect, imposed on
the privilege to conduct insurance business and not on the insurance business itself or
on the premiums paid under the said insurance policies. This means then that the
documentary stamp tax accrues when the said privilege is exercised. As the respondent
court stated, while it is true that a documentary stamp tax is levied on the document and
not on the property involved, the documentary stamp tax is not intended to be a tax on
the document alone. The law taxes the document because of the transaction so that the
tax becomes due and payable at the time the transaction is had or accomplished, in this
case, at the time of the issuance of the document.
Kenneth B. Minglana

COMMISSIONER OF INTERNAL REVENUE vs LINCOLN PHIL LIFE INSURANCE


379 SCRA 423 / March 19, 2002

FACTS:

Respondent Lincoln Phil. Issued a special kind of life insurance known as “Junior
Estate Builder Policy”, the distinguishing feature of which is its “automatic increase
clause” which provides for an automatic increase in the amount of life coverage upon
attainment by the insured of a certain age without the need of issuing a new policy. CIR
contends that while no new policy was issued, the original policy was essentially re-
issued when the additional obligation was assumed upon the effectivity of this
“automatic increase clause”. Therefore, the consequent increase is but a scheme to
avoid further tax obligations. As a result, CIR imposed documentary stamp tax on the
original policy and another documentary stamp tax on the same policy but upon the
effectivity of the automatic increase clause.

ISSUE:

WON the automatic increase clause should be considered as a new and


separate policy apart from the original?

RULING:

NO. Section 49 defines a policy as a written instrument where the contract of


insurance is set forth. Moreover, Section 50 provides that the policy may contain any
word, phrase, CLAUSE, … necessary to complete the contract of insurance. It is
therefore clear that any rider, clause or warranties attached to the policy is considered
part of the contract of insurance. Although the clause is to take effect in a latter time, it is
written into the policy at the time of issuance. The feature providing for an automatic
increase has already formed part of the insurance contract; hence there was no need
for execution of a new agreement for the increase in the coverage.
Sundae June A. Jugao

FGU INSURANCE vs. COURT OF APPEALS


454 SCRA 337

FACTS:

Anco Enterprises Company (ANCO), a partnership between Ang Gui and Co To,
was engaged in the shipping business. It owned the M/T ANCO tugboat and the D/B
Lucio barge which were operated as common carriers. Since the D/B Lucio had no
engine of its own, it could not maneuver by itself and had to be towed by a tugboat for it
to move from one place to another. On September 23, 1979, San Miguel
Corporation (SMC) shipped from Mandaue City, Cebu, on board the D/B Lucio,
for towage by M/T ANCO cases of pale pilsen and cerveza negra to consignee
SMC’s Sales Office in Iloilo and San Jose, Antique. The D/B Lucio was towed by
the M/T ANCO all the way from Mandaue City to San Jose, Antique. When the barge
and tugboat arrived at San Jose, Antique, the clouds over the area were dark and the
waves were already big. The arrastre workers unloading the cargoes of SMC on board
the D/B Lucio began to complain about their difficulty in unloading the cargoes. SMC’s
Supervisor, Fernando Macabuag, requested ANCO’s representative to transfer the
barge to a safer place because the vessel might not be able to withstand the big waves,
but it refused, so around the midnight, the barge sunk along
with 29,210 cases of Pale Pilsen and 500 cases of Cerveza Negra totalling
to P1,346,197. When SMC claimed against ANCO it stated that they agreed
that it would not be liable for any losses or damages resulting to the cargoes
by reason of fortuitous event and it was agreed to be insured with FGU for
20,000 cases or P858,500. ANCO filed against FGU but the latter alleged that
ANCO and SMC failed to exercise diligence of a good father of the family in
the care and supervision of the cargoes, thus it should be exempted from
liability.

ISSUE:

Whether or not FGU should be exempted from liability for the lost
cargoes because of negligence of ANCO.

RULING:

FGU should be exempted from liability. To be exempted from


responsibility, the natural disaster should have been the proximate and only
cause of the loss. There must have been no contributory negligence on the
part of the common carrier. In the case at bar, there was blatant negligence
on the part of M/T ANCO’s crew members, first in leaving the engine-
less barge D/B Lucio at the mercy of the storm without the assistance of the
tugboat, and again in failing to heed the request of SMC’s representatives to
have the barge transferred to a safer place. When evidence show that the
insured’s negligence or recklessness is so gross as to be sufficient to
constitute a willful act, the insurer must be exonerated. ANCO’s employees
are of such gross character that it amounts to a wrongful act which must
exonerate FGU from liability under the insurance contract.

Junyvil B. Tumbaga

GULF RESORTS INC. vs. PHILIPPINE CHARTER INSURANCE


CORPORATION
[G.R. No. 156167 May 16, 2005] 458 SCRA 550

FACTS:

Gulf Resorts is the owner of the Plaza Resort situated at Agoo, La Union
and had its properties in said resort insured originally with the American
Home Assurance Company (AHAC). In the first 4 policies issued, the risks of
loss from earthquake shock were extended only to petitioner’s two swimming
pools. Gulf Resorts agreed to insure with Phil Charter the properties covered
by the AHAC policy provided that the policy wording and rates in said policy
be copied in the policy to be issued by Phil Charter. Phil Charter issued Policy
to Gulf Resorts covering the period of March 14, 1990 to March 14, 1991 for
P10,700,600.00 for a total premium of P45,159.92. The break-down of
premiums shows that Gulf Resorts paid only P393.00 as premium against
earthquake shock (ES). In Policy No. 31944 issued by defendant, the shock
endorsement provided that “In consideration of the payment by the insured
to the company of the sum included additional premium the Company
agrees, notwithstanding what is stated in the printed conditions of this policy
due to the contrary, that this insurance covers loss or damage to shock to
any of the property insured by this Policy occasioned by or through or
inconsequence of earthquake. In Exhibit "7-C" the word" included" above the
underlined portion was deleted. On July 16, 1990 an earthquake struck
Central Luzon and Northern Luzon and plaintiff’s properties covered by Policy
No. 31944 issued by defendant, including the two swimming pools in its Agoo
Playa Resort were damaged. Petitioner advised respondent that it would be
making a claim under its Insurance Policy 31944 for damages on its
properties. Respondent denied petitioner’s claim on the ground that its
insurance policy only afforded earthquake shock coverage to the two
swimming pools of the resort. The trial court ruled in favor of respondent. In
its ruling, the schedule clearly shows that petitioner paid only a premium of
P393.00 against the peril of earthquake shock, the same premium it had paid
against earthquake shock only on the two swimming pools in all the policies issued by
AHAC.

ISSUE:

Whether or not the policy covers only the two swimming pools owned by Gulf
Resorts and does not extend to all properties damaged therein.

RULING:
Yes. All the provisions and riders taken and interpreted together,
indubitably show the intention of the parties to extend earthquake shock
coverage to the two swimming pools only. An insurance premium is the
consideration paid an insurer for undertaking to indemnify the insured
against a specified peril. In fire, casualty and marine insurance, the premium
becomes a debt as soon as the risk attaches. In the subject policy, no
premium payments were made with regard to earthquake shock coverage
except on the two swimming pools. There is no mention of any premium
payable for the other resort properties with regard to earthquake shock. This
is consistent with the history of petitioner’s insurance policies with AHAC.

William Z. Radaza

PHIL. CHARTER INSURANCE CORP. vs. CHEMOIL LIGHTERAGE CORP.


462 SCRA 77; June 29, 2005

FACTS:

A shipment of 62.06 metric tons of liquid chemical Dioctyl Phthalate (or


DOP) was shipped to consignee Plastic Group Phils., Inc. (or PGP) in Manila.
PGP insured the cargo with Philippine Charter Insurance Corporation as the
insurer. The cargo was unloaded to the tanker barge of Chemoil Lighterage
Corp., a common carrier, which transported the goods to Del Pan Bridge in
Pasig River. When the cargo was received by PGP, the chemical showed
discoloration from yellowish to amber, demonstrating that it was damaged.
PGP sought recovery from its insurer, Phil. Charter Insurance, which paid the
amount of the loss. PGP, thereafter, issued a Subrogation Receipt to the Phil.
Charter Insurance.

An action for damages was instituted by the petitioner-insurer (Phil.


Charter Insurance) against respondent-carrier (Chemoil). Respondent carrier
refused to pay the damages to the insurer, contending that the consignee
(PGP) failed to file any notice, claim or protest within the period required by
law which is a condition precedent to the accrual of a right of action against
the carrier. The carrier alleged that the telephone call made by a certain
employee of PGP, to one of the Vice Presidents of Chemoil, informing the
latter of the discoloration, is not the notice required by Article 366 of the
Code of Commerce.

ISSUE:

Whether or not a notice of claim filed with the carrier within the prescribe period
is indispensible in order that the insurer-subrogee can claim damages against the
carrier.

RULING:
A notice of claim to the carrier is indispensible before the insurer-subrogee can
recover from the carrier. Art. 366 of the Code of Commerce requires that a notice or
claim must be made against the carrier of the goods, in case of damage, within 24
hours following the receipt of the goods. The telephone call made by the PGP
(consignee of the goods) was not a substantial compliance to the required notice. Aside
from the telephone call, there was no other proof that a notice was relayed or filed with
the carrier immediately or within 24 hours from the time the goods were received. The
requirement that a notice of claim should be filed within the period stated by
Article 366 of the Code of Commerce is not an empty or worthless proviso.
The filing of a claim with the carrier within the time limitation therefore
actually constitutes a condition precedent to the accrual of a right of action
against a carrier for loss of, or damage to, the goods. The shipper or
consignee must allege and prove the fulfillment of the condition. If it fails to
do so, no right of action against the carrier can accrue in favor of the former.

Aurora Luanne R. Cembrano

SULPICIO LINES vs. FIRST LEPANTO


462 SCRA 125 / June 29, 2005

FACTS:

Delbros, Inc. (DELBROS) engaged the services of Sulpicio Lines, Inc. (SULPICIO) to
transport shipment of goods consisting of 3 wooden crates, from Cebu to Manila. The goods
were owned by Taiyo Yuden Philippines, Inc. (owner of the goods) and were covered by a
marine insurance policy issued by First Lepanto-Taisho Insurance Corporation (LEPANTO).

However, during the unloading at the pier in Manila, one crate fell. It was found to be
externally damaged and was no longer usable for their intended purpose hence it was sent back
to Cebu. The owner of the goods filed a claim with DELBROS for reimbursement. Upon the
latter’s refusal, the owner then sought payment from its insurer, LEPANTO. LEPANTO paid the
owner of the goods and then sought for reimbursement from DELBROS and SULPICIO, which
claims were subsequently denied.

LEPANTO alleges that payment to owner of the goods subrogated it to whatever right or
legal action the owner of the goods may have against DELBROS and SULPICIO. DELBROS in
its defense claims that assuming the contents of the crate in question were truly in bad order, the
fault is with SULPICIO which was responsible for the unloading of the crates. SULPICIO,
however, insists that it was only the external packaging that was damaged, and that there was no
actual damage to the goods such that would make them liable. According to it, damage to the
packaging is not tantamount to damage to the cargo.
ISSUE:

Whether or not Sulpicio Lines, Inc. is liable to the owner of the goods, and if so, to what
extent is its liability.

RULING:

Sulpicio Lines, Inc. is liable to the owner of the goods. The damage sustained by the
packaging of cargo while in SULPICIO’s custody resulted in its unfitness to be transported to
Singapore. Such failure to ship the cargo to its final destination because of the ruined packaging,
indeed, resulted in damages on the part of the owner of the goods.

As to its extent of liability, since insurer LEPANTO paid the owner of the goods under
the insurance policy, SULPICIO is liable to pay the amount paid by LEPANTO for the damages
sustained by the owner of the goods (P194,220.31).

Christine Mae P. Navarra

PHILIP S. YU vs. HON. COURT OF APPEALS, and VIVECA LIM YU


476 SCRA 443 / Nov. 29, 2005

FACTS:

Private respondent Viveca Lim Yu brought against her husband,


Petitioner Philip Sy Yu, an action for legal separation and dissolution of
conjugal partnership on the grounds of marital infidelity and physical abuse.
During trial,Viveca moved for the issuance of a subpoena
ducestecum and ad testificandum to certain officers of Insular Life Assurance
Co. Ltd. to compel production of the insurance policy and application of a
person suspected to be petitioner’s illegitimate child. The trial court denied
the motion. It ruled that the insurance contract is inadmissible evidence in
view of Circular Letter No. 11-2000, issued by the Insurance Commission
which presumably prevents insurance companies/agents from divulging
confidential and privileged information pertaining to insurance policies. It
added that the production of the application and insurance contract would
violate Article 280 of the Civil Code and Section 5 of the Civil Registry
Law, both of which prohibit the unauthorized identification of the parents of
an illegitimate child. Private respondent sought reconsideration of the Order,
but the motion was denied by the trial court.

Aggrieved, Viveca filed a petition for certiorari before the Court of


Appeals. It declared that petitioner’s objection to the admission of the
documents was premature, and the trial court’s pronouncement that the
documents are inadmissible, precipitate. Private respondent was merely
seeking the production of the insurance application and contract, and was
not yet offering the same as part of her evidence. Also, the contents of the
insurance application and insurance documents cannot be considered as
privileged information. Petitioner filed a motion for reconsideration but to no
avail. Hence, this appeals.

ISSUE:

Whether or not an application for insurance and an insurance policy


can be admissible as evidence
RULING:

The insurance application and the insurance policy were yet to be


presented in court, much less formally offered before it. In fact, private
respondent was merely asking for the issuance of subpoena
ducestecum and subpoena ad testificandum when the trial court issued the
assailed Order. Even assuming that the documents would eventually be
declared inadmissible, the trial court was not then in a position to make a
declaration to that effect at that point. Thus, it barred the production of the
subject documents prior to the assessment of its probable worth. Thus, in
declaring that the documents are irrelevant and inadmissible even before
they were formally offered, much less presented before it, the trial court
acted in excess of its discretion.

Anent the issue of whether the information contained in the documents


is privileged in nature, the same was clarified and settled by the Insurance
Commissioner’s opinion that the circular on which the trial court based its
ruling was not designed to obstruct lawful court orders. Hence, there is no
more impediment to presenting the insurance application and policy.

Albert G. Cong

DANZAS CORPORATION VS. ABROGAR


486 SCRA 80 (2005)

FACTS:

Danzas Corporation (Danzas) took a shipment of nine packages of ICS watches


for transport to Manila. The consignee, International Freeport Traders, Inc. (IFTI)
secured a marine risk note from Seaboard Eastern Insurance (Seaboard). On arrival via
Korean Airlines (KAL), and upon withdrawal, IFTI discovered that several watches were
missing. Seaboard, as insurer, paid the losses to IFTI, and invoking its right of
subrogation, filed a complaint against KAL. While the case was pending, IFTI accepted
the proposal of KAL to settle consignee’s claim and then signed a release form. Danzas
filed a motion to dismiss the case on the ground that Seaboard’s demand had been paid
or otherwise extinguished by KAL. The trial court issued an order denying the motion to
dismiss, hence the certiorari under Rule 65 before the Supreme Court.

ISSUE:
Was Seaboard’s right of subrogation extinguished when IFTI received payment
from KAL in settlement of its obligation?

RULING:

No. One of the many exceptions to the rule of subrogation is “if the assured by
his own act releases the wrongdoer or third party liable for the loss or damage from
liability, the insurer’s right of subrogation is defeated.” However, KAL, the wrongdoer,
was fully aware of the prior payment made by the insurer, Seaboard, to the consignee,
IFTI, evidencing bad faith during the time it made such settlement. Now there exists a
wealth of U.S. jurisprudence holding that whenever the wrongdoer settles with the
insured without the consent of the insurer and with knowledge of the insurer’s payment
and right of subrogation, such right is not defeated by the settlement.

Rogaciano A. Quico III

GAISANO CAGAYAN, INC. VS INSURANCE COMPANY OF NORTH AMERICA


490 SCRA 286 / June 8, 2006

FACTS:

Intercapitol Marketing Corporation (IMC) is the maker of Wrangler Blue Jeans.


while Levi Strauss (Phils.) Inc. (LSPI) is the local distributor of products bearing
trademarks owned by Levi Strauss & Co.

IMC and LSPI separately obtained from Insurance Company of North America
fire insurance policies for their book debt endorsements related to their ready-made
clothing materials which have been sold or delivered to various customers and dealers
of the Insured anywhere in the Philippines which are unpaid 45 days after the time of
the loss. On February 25, 1991, Gaisano Superstore Complex in Cagayan de Oro City,
containing the ready-made clothing materials sold and delivered by IMC and LSPI was
consumed by fire.
Insurance Company of North America filed a complaint for damages against
Gaisano Cagayan, Inc. alleges that IMC and LSPI filed their claims under their
respective fire insurance policies which it paid thus it was subrogated to their rights.
Petitioner argued that it cannot be held liable because it was destroyed due to fortuities
event or force majeure. RTC decided that IMC and LSPI retained ownership of the
delivered goods until fully paid, it must bear the loss (res perit domino). CA Reversed
saying that sales invoices is an exception under Article 1504 (1) of the Civil Code to res
perit domino.

ISSUE:

Whether or not Insurance Company of North America can claim against Gaisano
Cagayan for the debt that was insured.

RULING:

YES. Petition is partly GRANTED the order to pay P535,613 is DELETED

Insurance policy is clear that the subject of the insurance is the book debts and
NOT goods sold and delivered to the customers and dealers of the insured. IMC and
LSPI did not lose complete interest over the goods. They have an insurable interest until
full payment of the value of the delivered goods. Unlike the civil law concept of res perit
domino, where ownership is the basis for consideration of who bears the risk of loss, in
property insurance, one's interest is not determined by concept of title, but whether
insured has substantial economic interest in the property. Section 13 of our Insurance
Code defines insurable interest as "every interest in property, whether real or personal,
or any relation thereto, or liability in respect thereof, of such nature that a contemplated
peril might directly damnify the insured." Parenthetically, under Section 14 of the same
Code, an insurable interest in property may consist in: (a) an existing interest; (b) an
inchoate interest founded on existing interest; or (c) an expectancy, coupled with an
existing interest in that out of which the expectancy arises.

Insurance in this case is not for loss of goods by fire but for petitioner's accounts
with IMC and LSPI that remained unpaid 45 days after the fire - obligation is pecuniary
in nature. Obligor should be held exempt from liability when the loss occurs thru a
fortuitous event only holds true when the obligation consists in the delivery of a
determinate thing and there is no stipulation holding him liable even in case of fortuitous
event.

The subrogation receipt, by itself, is sufficient to establish not only the


relationship of respondent as insurer and IMC as the insured, but also the amount paid
to settle the insurance claim. Under Art. 2207, If the plaintiff's property has been
insured, and he has received indemnity from the insurance company for the injury or
loss arising out of the wrong or breach of contract complained of, the insurance
company shall be subrogated to the rights of the insured against the wrongdoer or the
person who has violated the contract.

As to LSPI, no subrogation receipt was offered in evidence. Failure to


substantiate the claim of subrogation is fatal to petitioner's case for recovery of the
amount of P535,613.
Mary Christine Anthonette M. Salise-Punzalan

PRUDENTIAL GUARANTEE & ASSURANCE vs. TRANS-ASIA


491 SCRA 411 / June 20, 2006

FACTS:

TRANS-Asia is the owner of the vessel M/V Asia Korea. PRUDENTIAL insured
M/V Asia Korea for loss/damage of the hull and machinery arising from perils, inter alia,
of fire and explosion. This is evidenced by Marine Policy No. MH93/1363 . While the
policy was in force, a fire broke out while [M/V Asia Korea was] undergoing repairs at
the port of Cebu which prompted TRANS-ASIA to file its notice of claim for damage
sustained by the vessel with a reservation of its right to subsequently notify Prudential to
the full amount of the claim upon final survey and determination by average adjuster of
the damage sustain by reason of fire. Later, TRANS-Asia executed a document
denominated “ Loan and Trust Receipt” a loan without interest under the Policy availed
by Trans-asia , repayable only in the event and to the extent that any net recovery is
made by the latter. After the ‘loan’ was released Prudential sent a letter to TRANS-ASIA
stating that the former’s claim under the insurance is denied for having been in breach
of policy conditions, among them “ WARRANTED VESSEL CLASSED AND CLASS
MAINTAINED” and that the claim is not compensable and demanding the return of the
amount released under loan.

It interpreted the provision to mean that TRANS-ASIA is required to maintain the


vessel at a certain class at all times pertinent during the life of the policy. According to
the court a quo, TRANS-ASIA failed to prove compliance of the terms of the warranty,
the violation thereof entitled PRUDENTIAL, the insured party, to rescind the contract.

ISSUE:

Whether TRANS-ASIA is entitled to claim under the policy.

RULING:

Yes. The supreme court find that the Court of Appeals was in no error when it held that
PRUDENTIAL, in renewing TRANS-ASIA’s insurance policy for two consecutive years after
the loss covered by Policy No. MH93/1363, was considered to have waived TRANS-ASIA’s
breach of the subject warranty, if any. Breach of a warranty or of a condition renders the contract
defeasible at the option of the insurer; but if he so elects, he may waive his privilege and power
to rescind by the mere expression of an intention so to do. In that event his liability under the
policy continues as before. There can be no clearer intention of the waiver of the alleged breach
than the renewal of the policy insurance granted by PRUDENTIAL to TRANS-ASIA in
MH94/1595 and MH95/1788, issued in the years 1994 and 1995, respectively.

Ropa Arienza- Arpilleda


CIR vs. PHILHEALTH CARE PROVIDERS INC.
April 24, 2007

FACTS:

The Philippine Health Care Providers, Inc., herein respondent, is a corporation organized
to establish, maintain, conduct and operate a prepaid group practice health care delivery system
or a health maintenance organization to take care of the sick and disabled persons enrolled in the
health care plan and to provide for the administrative, legal, and financial responsibilities of the
organization. On December 10, 1987, respondent wrote the Commissioner of Internal Revenue
(CIR), petitioner, inquiring whether the services it provides to the participants in its health care
program are exempt from the payment of the VAT. Petitioner CIR, through the VAT Review
Committee of the Bureau of Internal Revenue (BIR), issued VAT Ruling No. 231-88 stating that
respondent, as a provider of medical services, is exempt from the VAT coverage. This Ruling was
subsequently confirmed by Regional Director Osmundo G. Umali of Revenue Region No. 8. On
January 27, 2000, petitioner CIR sent respondent a letter demanding payment of "deficiency
VAT" DST in the amount of P224,702,641.18 for taxable years 1996 and 1997. Respondent filed
protest questioning the assessment notices with the Court of Tax Appeals (CTA) to which the
latter partially granted the petition for review and ordered respondent to pay the deficiency VAT
and petitioner is ordered to desist from collecting the said DST.

ISSUE:

Whether respondent's services are subject to VAT.

RULING:

Section 103 of the the National Internal Revenue Code of 1977, as amended by E.O. No.
273 (VAT Law) and R.A. No. 7716 (E-VAT Law specifies the exempt transactions from the
provision of Section 102, thus:

SEC.103. Exempt Transactions. - The following shall be exempt from the value-added
tax:
(l) Medical, dental, hospital and veterinary services except those rendered by
professionals.

The import of the above provision is plain as it contemplates the exemption from VAT of
taxpayers engaged in the performance of medical, dental, hospital, and veterinary services. In its
letter to the BIR requesting confirmation of its VAT-exempt status, respondent described its
services as a prepaid group practice health care delivery system where individuals enrolled in
Health Care's health care program are entitled to preventive, diagnostic, and corrective medical
services to be dispensed by Health Care's duly licensed physicians, specialists, and other
professional technical staff. To be entitled to receive such medical services from Health Care, an
individual must enroll in Health Care's health care program and pay an annual fee. Enrollment in
Health Care's health care program is on a year-to-year basis and enrollees are issued
identification cards.

Perforce, as respondent does not actually provide medical and/or hospital


services, as provided under Section 103 on exempt transactions, but merely
arranges for the same, its services are not VAT-exempt.
Silhouette B. Adobas

Ong Lim Sing , Jr. FEB Leasing & Finance Corp


524 SCRA 333 / 8 June 2007

FACTS:

On March 9, 1995, FEB Leasing and Finance Corporation (FEB) entered


into a lease of equipment and motor vehicles with JVL Food Products (JVL).
On the same date, Vicente Ong Lim Sing, Jr. (Lim) executed an Individual
Guaranty Agreement with FEB to guarantee the prompt and faithful
performance of the terms and conditions of the aforesaid lease agreement.
Corresponding Lease Schedules with Delivery and Acceptance
Certificates over the equipment and motor vehicles formed part of the
agreement. Under the contract, JVL was obliged to pay FEB an aggregate
gross monthly rental of One Hundred Seventy Thousand Four Hundred
Ninety-Four Pesos (P170,494.00).

JVL defaulted in the payment of the monthly rentals. As of July 31,


2000, the amount in arrears, including penalty charges and insurance
premiums, amounted to Three Million Four Hundred Fourteen Thousand Four
Hundred Sixty-Eight and 75/100 Pesos (P3,414,468.75). On August 23, 2000,
FEB sent a letter to JVL demanding payment of the said amount. However,
JVL failed to pay.

On December 6, 2000, FEB filed a Complaint with the Regional Trial


Court of Manila, docketed as Civil Case No. 00-99451, for sum of money,
damages, and replevin against JVL, Lim, and John Doe.

ISSUE:

Does a lessee have an insurable interest in the equipments?

RULING:

The stipulation in Section 14 of the lease contract, that the equipment


shall be insured at the cost and expense of the lessee against loss, damage,
or destruction from fire, theft, accident, or other insurable risk for the full
term of the lease, is a binding and valid stipulation. Petitioner, as a lessee,
has an insurable interest in the equipment and motor vehicles
leased. Section 17 of the Insurance Code provides that the measure of
an insurable interest in property is the extent to which the insured might be
damnified by loss or injury thereof. It cannot be denied that JVL will be
directly damnified in case of loss, damage, or destruction of any of the
properties leased.
Kenneth Minglana

ETERNAL GARDENS VS PHIL. AMERICAN LIFE

FACTS

Eternal and PhilAm Life entered into a Group Insurance Policy wherein those who
purchase burial lots from Eternal Gardens automatically becomes one of the insured in the said
group insurance policy. One purchaser of the burial lots named Jhon Chuang died. Phil Am Life
filed a claim for the proceeds. The claim was denied on the ground that the application of Jhon
Chuang, sent more than a year ago, was not yet approved by Phil Am Life. Insurer further avers
that mere acceptance of the premium paid by Eternal did not mean that said application was
already approved.

ISSUE

May the inaction of the insurer on the insurance application be considered as an


approval of the application?

RULING
Yes. mere inaction of the insurer on the insurance application must not work to prejudice the
insured; it cannot be interpreted as a termination of the insurance contract. The termination of the
insurance contract by the insurer must be explicit and unambiguous.In order to protect the interest of
insurance applicants, insurance companies must be obligated to act with haste upon insurance
applications, to either deny or approve the same, or otherwise be bound to honor the application as a
valid, binding, and effective insurance contract.
Sundae June A. Jugao

LALICAN vs. THE INSULAR LIFE CO.


597 SCRA 159

FACTS:

Eulogio Lalican applied for insurance with Insular Life thru the latter’s agent,
Malaluan with his wife Violeta as the beneficiary. A policy was issued in his favor. It is a
20-year endowment plan with the total value of 1.5M. The premiums are payable on a
quarterly basis until the end of the 20 year period. He failed to pay on the due date for
the third quarter and also was not able to tender payment after the lapse of 31 days
grace period. Consequently his policy has lapsed and avoided by non-payment of
premium. Eulogio went to the residence of the agent Malaluan and applied for
reinstatement of the lapsed policy. He gave all the requirements including the amount
which is enough to cover all the unpaid premiums and interests. Unfortunately, on the
same day of his application for reinstatement and few hours thereafter, Eulogio died of
cardiac arrest. After a few days, Violeta filed a claim of payment of full proceeds of the
policy. Insular Life refused her claim saying that said policy has lapsed and Elugio has
failed to comply with the requirements of reinstatement one of which states “policy is
reinstated upon the approval of the company during the lifetime and Good health of
Eulogio.” Violeta sued to recover the amount with the RTC. One of her principal
contention is that as stated under Section 19 of the Insurance Code, Insurable interest
in Life Insurance policies need not exist in the time of loss. RTC ruled in favor of Insular
Life holding that Violeta cannot recover from a lapsed and void policy and that mere
payment to the agent does not result into an automatic reinstatement.

ISSUE:

May Violeta recover the proceeds from the policy?

RULING:

No. The death of Eulogio has made it impossible for him to comply with the
conditions of reinstatement. The privilege of reinstatement does not give the insured an
absolute right to reinstatement by mere filing of application. The insurer has the right to
deny reinstatement if it is not satisfied as to the insurability of the insured. Moreover,
after the death of the insured, the Insurance Company cannot be compelled to accept
application for reinstatement since condition precedent for reinstatement can no longer
be determined or satisfied.
Junyvil B. Tumbaga

RO Q UE v. I A C (PI ON EE R I NS U R A NC E A N D SURETY CORP.)


139 SCRA 596 November 11, 1985

FACTS:

On February 19, 1972, common carrier Manila Bay Lighterage


Corp. entered into a contract with Roque Timber Enterprises and Chiong. The
contract stated that Manila Bay would carry 422.18 cu. meters of logs on its
vessel Mable 10 from Malampaya Sound, Palawan to Manila North Harbor. Roque
insured the logs with Pioneer Insurance for P100,000. O n Fe b r u a r y 2 9 , 1 9 7 2 , 8 11
l o g s we re l o a d e d i n Malampaya but en route to Manila, Mable 10 sank.
Consequently, Roque and Chiong wrote a letter to Manila Bay, demanding payment of
P150,000.00 for th e l o s s o f t h e s h i p m e n t p l u s P 1 0 0 ,0 0 0 . 0 0 a s unrealized
profits but the latter ignored the demand. A letter was also sent to Pioneer, claiming the
full amount of P100,000.00 under the insurance policy but Pioneer refused to
pay on the ground that its liability depended upon the "Total Loss by Total Loss of
Vessel only".- Af t e r h e a r i n g , t h e t ri a l c o u r t f a vo re d R o q u e . Pioneer and
Manila Bay were ordered to pay RoqueP100,000. Pioneer appealed the decision
wherein Pioneer was absolved from l i a b i l i t y a ft e r f i n d i n g t h a t th e re wa s a
b re a ch o f implied warranty of seaworthiness on the part of the petitioners and that the
loss of the insured cargo was caused by the "perils of the ship" and not by the "
p e ri l s o f th e se a " . I t ru l e d t h a t t h e l o s s i s n o t covered by the marine
insurance policy. It was alleged that Mable 10 was not seaworthy and that it
developed a leak. Petitioners contend that the implied warranty
of seaworthiness provided for in the Insurance Code refers only to the
responsibility of the shipowner who must see to it that his ship is reasonably fit to make
in safety the contemplated voyage. The petitioners state that a mere shipper of cargo,
having no control over the ship, has nothing to do wi t h i t s se a wo r t h i n e s s.
T h e y a rg u e th a t a ca r g o owner has no control over the structure of the ship, its
cables, anchors, fuel and provisions, the manner of loading his cargo and the cargo of
other shippers, and the hiring of a sufficient number of competent officers and
seamen.

ISSUE:

W he t he r o r no t th e lo s s s h ou l d ha ve b e e n c ov e re d b y th e
marine insurance policy.

RULING:

No. It is universally accepted that in every contract of insurance


upon anything which is the subject of marine insurance, a warranty is
implied that the ship shall be seaworthy at the time of the inception of the
voyage. In marine insurance, the risks insured against are classifi ed
as 'perils of the sea,’ which includes such losses that are of extraordinary
nature, o r a r is e fro m s om e ov e r w he lm i ng p ow e r , wh ic h cannot be
guarded against by the ordinary exertion of human skill and prudence.
Based on Sec. 113 and Sec. 99 of the Insurance Code, the term "cargo"
can be the subject of marine insurance and that once it is so made, the
implied warranty of seaworthiness immediately attaches to whoever
is insuring the cargo whether he be the shipowner or not. The fact that
the un-seaworthiness of the ship was unknown to the insured is
immaterial in ordinary marine insurance and may not be used by him as a
defense in order to recover on the marine insurance policy. Since the law
provides for an implied warranty of seaworthiness in every contract of
ordinary marine in s u ra nc e , it b e c om e s th e o b l ig a t i on o f a ca rg o
owner to look for a reliable common carrier which keeps its vessels in
seaworthy condition. The shipper of cargo may have no control over the
vessel but he has full control in the choice of the common carrier that will
transport his goods.- In ma r in e ca s e s , t he r is k s in s u re d a ga in s t a re
‘perils of the sea.’ The term extends only to losses caused by sea damage,
or by the violence of the elements, and does not embrace all losses
happening at sea. It is quite unmistakable that the loss of the cargo
was due to the perils of the ship rather than the perils of the sea. Los s
w hi c h , i n th e o rd in a r y co u rs e o f e v e n ts , results from the natural and
inevitable action of the sea, from the ordinary wear and tear of the ship, or
from the negligent failure of the ship's owner to provide the vessel with
proper equipment to convey the cargo under ordinary conditions, is not a
‘peril of the sea’ but is called ‘peril of the ship.’
William Z. Radaza

CATHAY INSURANCE CO. vs. CA


151 SCRA 710; June 30, 1987

FACTS:

Remington Industrial Sales Corp. was the consignee of a shipment of seamless


steel pipes which was covered by a marine insurance policy issued by Cathay
Insurance. When the shipment arrived, it was found that the steel pipes were heavily
rusted. Thus, Remington sought to claim the amount of the insurance for the damage of
the shipment, but Cathay refused to pay on the ground that the rusting of the pipes was
not a risk insured against in the policy.

Remington filed a complaint against Cathay Insurance seeking to collect the


amount of the insurance representing the losses or damages incurred in the shipment. It
alleged that, although rusting of the steel pipes was not clearly indicated in the terms of
the policy, nevertheless, it is impliedly covered under the marine insurance as part of
the “perils of the sea.”

ISSUE:

Whether or not rusting of steel pipes is a risk covered under a marine insurance
although not clearly indicated in the terms of the policy.

RULING:

Rusting of the steel pipes is a risk covered under a marine insurance. There is no
question that the rusting of steel pipes in the course of a voyage is a "peril of the sea" in
view of the toll on the cargo of wind, water, and salt conditions. At any rate if the insurer
cannot be held accountable therefor, it would result in the failure to observe a cardinal
rule in the interpretation of contracts, namely, that any ambiguity therein should be
construed against the maker/issuer/drafter thereof, which is the insurer. Besides, the
precise purpose of insuring cargo during a voyage would be rendered fruitless.
Aurora Luanne R. Cembrano

THE PHILIPPINE AMERICAN GENERAL INSURANCE COMPANY, INC. vs. CA


273 SCRA 262 / June 11, 1997

FACTS:

MV Asilda is a vessel owned and operated by private respondent Felman Shipping Lines
(FELMAN). Coca-Cola Bottlers Philippines, Inc. (Coca-Cola Bottlers) loaded on board MV
Asilda 7,500 cases of 1-liter Coca-Cola softdrink bottles. The shipment was insured with
petitioner Philippine American General Insurance Co., Inc. (PHILAMGEN).

The day after the vessel left the port, it capsized and sank bringing down her entire cargo
including the 7,500 cases of softdrink bottles.

Because FELMAN refused to reimburse Coca-Cola Bottlers for damages, the latter filed
an insurance claim with PHILAMGEN which paid its claim of P755,250.00. PHILAMGEN then
sought recourse against respondent FELMAN claiming its right of subrogation.

FELMAN refused to pay PHILAMGEN alleging that there was actually no right of
subrogation transmitted. The reason is that Coca-Cola Bottlers had breached its implied warranty
on the vessel’s seaworthiness for being top-heavy as 2,500 cases of the softdrink bottles were
improperly stowed on deck. Hence, when PHILAMGEN paid the claim of the bottling firm there
was in effect a “voluntary payment” and no right of subrogation accrued in its favor. Moreover,
FELMAN contends that it had abandoned its liability under Art. 587 of the Code of Commerce.

ISSUES:

1. Whether or not MV Asilda was seaworthy when it left the port of origin.
2. Whether or not the limited liability or the right of abandonment under Art. 587 of the
Code of Commerce should apply.
3. Whether or not PHILAMGEN was properly subrogated to the rights and legal actions
which the shipper had against FELMAN, the shipowner.

RULING:
1. MV Asilda was unseaworthy when it left the port of origin. The distribution of the cargo on
board was done in such a manner that the vessel was in top heavy condition which rendered
her unstable and unseaworthy for that particular voyage. It is settled that carrying a deck
cargo raises the presumption of unseaworthiness unless it can be shown that the deck cargo
will not interfere with the proper management of the ship. However, in this case MV Asilda
was not designed to carry substantial amount of cargo on decl. the inordinate loading of
cargo deck resulted in the decrease of the vessel’s metacentric height this making it unstable.

2. Art. 587 of the Code of Commerce is not applicable to the case at bar. Under said article,
liability of the shipowner can be limited through abandonment of the vessel, its equipment
and freightage. However, this rule does not apply to cases where the injury or average was
occasioned by the shipowner’s own fault. Thus, Art. 587 applies only to situations where the
fault or negligence is committed solely by the captain but not those committed by the
shipowner.
In the case at bar, closer supervision on the part of the shipowner could have prevented the
fatal miscalculation and as such, FELMAN was equally negligent. It cannot therefore escape
liability by virtue of Art. 587 of the Code of Commerce.

3. It is generally held that in every marine insurance policy the assured impliedly warrants to
the assurer that the vessel is seaworthy and such warranty is as much a term of the contract as
if expressly written on the face of the policy. However, an insertion of waiver clauses in
cargo policies is in recognition of the realistic fact that cargo owners cannot control the state
of the vessel. PHILAMGEN expressly admitted the warranty of seaworthiness in the
insurance policy it issued. Thus it can be said that with such categorical waiver,
PHILAMGEN has accepted the risk of unseaworthiness so that if the ship should sink by
unseaworthiness, as what occurred in this case, PHILAMGEN is liable.

Therefore, the payment made by PHILAMGEN to Coca-Cola Bottlers gave the


former the right to bring an action as subrogee against FELMAN.
Christine Mae P. Navarra

MAYER STEEL PIPE CORPORATION vs. CA


G.R. No. 124050 / June 19, 1997

FACTS:

Petitioner Hongkong Government Supplies Department (Hongkong) contracted petitioner


Mayer Steel Pipe Corporation (Mayer) to manufacture and supply various steel pipes and
fittings.Prior to the shipping, petitioner Mayer insured the pipes and fittings against all risks with
private respondents South Sea Surety and Insurance Co., Inc. (South Sea) and Charter Insurance
Corp. (Charter). Petitioners Mayer and Hongkong jointly appointed Industrial Inspection
(International) Inc. as third-party inspector to examine whether the pipes and fittings are
manufactured in accordance with the specifications in the contract. When the goods reached
Hongkong, it was discovered that a substantial portion thereof was damaged. Thereby,
Petitioners filed a claim against private respondents for indemnity under the insurance contract.
But, private respondents refused to pay because the insurance surveyor's report allegedly showed
that the damage is a factory defect. Hence, petitioners filed an action against private respondents
to recover the sum.

The trial court ruled in favour of petitioners. It found that the damage to the goods is not
due to manufacturing defects.It also noted that the insurance contracts executed by petitioner
Mayer and private respondents are "all risks" policies which insure against all causes of
conceivable loss or damage. The only exceptions are those excluded in the policy, or those
sustained due to fraud or intentional misconduct on the part of the insured. Private respondents
elevated the case to respondent Court of Appeals. CA affirmed. However, it set aside the decision
of the trial court and dismissed the complaint on the ground of prescription. It held that the action
is barred under Section 3(6) of the Carriage of Goods by Sea Act since it was filed only on April
17, 1986, more than two years from the time the goods were unloaded from the vessel.

ISSUE:

Whether or not cause of action had already prescribed

RULING:
The cause of action has not yet prescribed. Section 3(6) of the Carriage of Goods by Sea
Act states that the carrier and the ship shall be discharged from all liability for loss or damage to
the goods if no suit is filed within one year after delivery of the goods or the date when they
should have been delivered. Under this provision, only the carrier's liability is extinguished if no
suit is brought within one year. But the liability of the insurer is not extinguished because the
insurer's liability is based not on the contract of carriage but on the contract of insurance. The
Carriage of Goods by Sea Act governs the relationship between the carrier on the one hand and
the shipper, the consignee and/or the insurer on the other hand. It defines the obligations of the
carrier under the contract of carriage. It does not, however, affect the relationship between the
shipper and the insurer. The latter case is governed by the Insurance Code. An insurance contract
is a contract whereby one party, for a consideration known as the premium, agrees to indemnify
another for loss or damage which he may suffer from a specified peril. An "all risks" insurance
policy covers all kinds of loss other than those due to willful and fraudulent act of the
insured. Thus, when private respondents issued the "all risks" policies to petitioner Mayer, they
bound themselves to indemnify the latter in case of loss or damage to the goods insured. Such
obligation prescribes in ten years, in accordance with Article 1144 of the New Civil Code.
Albert G. Cong

PHILIPPINE HOME ASSURANCE CORP. vs. Court of appeals


257 SCRA 468 (1996)

FACTS:

Eastern Shipping Lines, Inc. (ESLI) loaded on board SS Eastern Explorer in


Kobe, Japan, several shipments bound to Manila and Cebu. While the vessel is off
Okinawa, Japan, a small flame on the acetylene cylinder in the main deck level set
ablaze the whole superstructure of the vessel. Thereafter, SS Eastern Explorer was
found to be constructive total loss and its voyage declared abandoned. A tugboat under
the control of Fukuda Salvage Co. arrived near the vessel and towed the vessel for the
port of Naha, Japan.

After the fire was extinguished, the cargoes which were saved were loaded to
another vessel for delivery for their original of port of destination. ESLI charged the
consignees several amounts corresponding to additional freight and salvage charges.
The charges were all paid by Philippine Home Assurance Corporation (PHAC) under
protest for and in behalf of the consignees. PHAC, as subrogee of the consignees,
thereafter filed a complaint before the RTC against ESLI to recover said sum on the
ground that the same were actually damages directly brought about by the fault,
negligence, illegal act and/or breach of contract of ESLI. The trial court dismissed the
PHAC’s complaint and ruled in favor of ESLI.

ISSUE:

Were the expenses incurred in saving the cargo considered general average?

RULING:

No. As a rule, general or gross averages include all damages and expenses
which are deliberately caused in order to save vessels, its cargo or both at the same
time, from a real and known risk. While the instant case may technically fall within the
purview of the said provision, the formalities prescribed under Article 813 and 814 of the
Code of Commerce in order to incur the expenses and cause the damage
corresponding to gross average were not complied with. Consequently, ESLI’s claim for
contribution from the consignees of the cargo at the time of the occurrence of the
average turns to naught.

Rogaciano A. Quico III

INTERNATIONAL CONTAINER TERMINAL SERVICES, INC.,


vs. FGU INSURANCE CORPORATION
556 SCRA 194 (2008)

FACTS:

In a Decision dated June 27, 2008, the Court denied the petition filed in this case
and affirmed the CA Decision dated October 22, 2003 and Resolution dated January 8,
2004, finding petitioner liable for the full amount of the shipment which was lost while in
its charge. Petitioner filed a motion for reconsideration, which was denied by the Court
with finality per Resolution dated August 27, 2008.Undaunted, petitioner filed the
present second motion for partial reconsideration where it solely assails the award and
reckoning date of the 12% interest imposed by the RTC on it adjudged liability.
Petitioner contends that the complaint filed before the RTC is not one for loan or
forbearance of money, but one for breach of contract or damages; hence, petitioner
insists that the interest rate should be the legal rate of 6%, and not 12%. Petitioner also
argues that the RTC reckoned the date when interest should accrue on the date when
respondent FGU Insurance Corporation paid the amount insured, or on January 3,
1995. Petitioner contends that this is erroneous and the date should be reckoned from
the time when respondent filed the complaint with the RTC, which is on April 10, 1995.

ISSUES:

Is the 12 percent interest rate valid? When shall be the reckoning date for the
interest rate to accrue?

RULING:

No. The interest rate of 6% should have been imposed and not 12%, as affirmed by
the Court. Also, it should have been reckoned from April 10, 1995, when respondent
filed by the complaint for sum of money, and not January 3, 1995, which was the date
respondent paid the amount insured to the Republic Asahi Glass Corporation (RAGC).
The claim in this case is one for reimbursement of the sum of money paid by FGU
Insurance Corporation to RAGC. This is not one for forbearance of money, goods or
credit. Forbearance in the context of the usury law is a contractual obligation of lender
or creditor to refrain, during a given period of time, from requiring the borrower or debtor
to repay a loan or debt then due and payable.[1] Thus the interest rate should be as it
is hereby fixed at 6%. Moreover, the interest rate of 6% shall be computed from the
date of filing of the complaint, i.e., April 10, 1995. This is in accordance with the ruling
that where the demand cannot be established with reasonable certainty, the interest
shall begin to run only from the date the judgment of the court is made (at which time
the quantification of damages may be deemed to have been reasonably ascertained).

Mary Christine Anthonette M. Salise-Punzalan

ABOITIZ SHIPPING vs. Ins. Co. of NORTH AMERICA


561 SCRA 262 (2008)

FACTS:

On June 20, 1993, MSAS Cargo International Limited and/or Associated and/or
Subsidiary Companies (MSAS) procured a marine insurance policy from respondent
ICNA UK Limited of London. The insurance was for a transshipment of a certain
wooden work tools and workbenches purchased for the consignee Science Teaching
Improvement Project (STIP). Ecotech Center, Sudlon, Lahug, Cebu City, Philippines.

ICNA issued an “all-risk” open marine policy, stating: This Company, in


consideration of a premium as agreed and subject to the terms and conditions printed
hereon, does insure for MSAS Cargo International Limited &/or Associated &/or
Subsidiary Companies on behalf of the title holder: -Loss, if any, payable to the Assured
or order.

The cargo, packed inside one container van, was shipped “freight prepaid” from
Hamburg, Germany on board M/S Katsuragi. A clean bill of lading was issued by
Hapag-Lloyd which stated the consignee to be STIP, Ecotech Center, Sudlon, Lahug,
Cebu City.

The container van was then off-loaded at Singapore and transshipped on board
M/S Vigour Singapore. On July 18, 1993, the ship arrived and docked at the Manila
International Container Port where the container van was again off-loaded.

On July 26, 1993, the cargo was received by petitioner Aboitiz Shipping
Corporation (Aboitiz) through its duly authorized booking representative, Aboitiz
Transport System. The bill of lading issued by Aboitiz contained the notation “grounded
outside warehouse.”

The container van was stripped and transferred to another crate/container van
without any notation on the condition of the cargo on the Stuffing/Stripping Report. On
August 1, 1993, the container van was loaded on board petitioner’s vessel, MV Super
Concarrier I. The vessel left Manila en route to Cebu City on August 2, 1993.

On August 3, 1993, the shipment arrived in Cebu City and discharged onto a
receiving apron of the Cebu International Port. It was then brought to the Cebu Bonded
Warehousing Corporation pending clearance from the Customs authorities.

In the stripping report dated August 5, 1993, petitioner’s checker noted that the
crates were slightly broken or cracked at the bottom. On August 11, 1993, the cargo
was withdrawn by the representative of the consignee, Science Teaching Improvement
Project (STIP) and delivered to Don Bosco Technical High School, Punta Princesa,
Cebu City. Mayo B. Perez, then Claims Head of petitioner, received a telephone call
from Willig informing that the cargo sustained water damage. In a letter, Willig informed
Aboitiz of the damage noticed upon the opening of the cargo. The letter stated that the
crate was broken at its bottom part such that the contents were exposed: the work tools
and workbenches were found to have been completely soaked in water with most of the
packing cartons already disintegrating. The crate was properly sealed off from the
inside with tarpaper sheets. On the outside, galvanized metal bands were nailed onto
all the edges. The letter concluded that apparently, the damage was caused by water
entering through the broken parts of the crate.

On September 21, 1993, the consignee filed a formal claim with Aboitiz for the
damaged condition of the goods. Aboitiz refused to settle the claim. On October 4,
1993, ICNA paid the amount of P280,176.92 to consignee. A subrogation receipt
executed in its favor. Despite follow-ups, however, no reply was received from Aboitiz.

ICNA filed a civil complaint against Aboitiz for collection of actual damages.
ICNA alleged that the damage sustained by the shipment was exclusively and solely
brought about by the fault and negligence of Aboitiz when the shipment was left
grounded outside its warehouse prior to delivery. Aboitiz disavowed any liability and
asserted that the claim had no factual and legal bases. It countered that the complaint
stated no cause of action, plaintiff ICNA had no personality to institute the suit, the
cause of action was barred, and the suit was premature there being no claim made
upon Aboitiz.

On November 14, 2003, the RTC rendered judgment against ICNA: the RTC
ruled that ICNA failed to prove that it is the real-party-in-interest to pursue the claim
against Aboitiz. ICNA appealed to the CA and the latter reversed and set aside the
decision of the RTC contending that the right of subrogation accrues simply upon
payment by the insurance company of the insurance claim. As subrogee, ICNA is
entitled to reimbursement from Aboitiz, even assuming that it is an unlicensed foreign
corporation.

The CA ruled that the presumption that the carrier was at fault or that it acted
negligently was not overcome by any countervailing evidence. Hence, the trial court
erred in dismissing the complaint and in not finding that based on the evidence on
record and relevant provisions of law, Aboitiz is liable for the loss or damage sustained
by the subject cargo.

ISSUE:

W/N ICNA can claim under the right of subrogation.

RULING:

YES. CA affirmed.
 Only when that foreign corporation is "transacting" or "doing business" in
the country will a license be necessary before it can institute suits. It
may, however, bring suits on isolated business transactions, which is not
prohibited under Philippine law
 The policy benefits any subsequent assignee, or holder, including the
consignee, who may file claims on behalf of the assured.

Sec. 57 of the Insurance Code provides :A policy may be so framed that it


will inure to the benefit of whomsoever, during the continuance of the risk,
may become the owner of the interest insured.

Art. 2207 of the Civil Code provides: If the plaintiff's property has been
insured, and he has received indemnity from the insurance company for the
injury or loss arising out of the wrong or breach of contract complained of,
the insurance company shall be subrogated to the rights of the insured
against the wrongdoer or the person who has violated the contract. If the
amount paid by the insurance company does not fully cover the injury or
loss, the aggrieved party shall be entitled to recover the deficiency from the
person causing the loss or injury.

 This right of subrogation, however, has its limitations.


o First, both the insurer and the consignee are bound by the contractual
stipulations under the bill of lading
o Second, the insurer can be subrogated only to the rights as the insured
may have against the wrongdoer. If by its own acts after receiving
payment from the insurer, the insured releases the wrongdoer who
caused the loss from liability, the insurer loses its claim against the latter.

Article 366 of the Civil Code provides: Within twenty four hours following the
receipt of the merchandise, the claim against the carrier for damages or
average which may be found therein upon opening the packages, may be
made, provided that the indications of the damage or average which give
rise to the claim cannot be ascertained from the outside part of such
packages, in which case the claim shall be admitted only at the time of
receipt.
After the periods mentioned have elapsed, or the transportation charges
have been paid, no claim shall be admitted against the carrier with regard to
the condition in which the goods transported were delivered.

 The call was made 2 from delivery, a reasonable period considering that
the goods could not have corroded instantly overnight such that it could
only have sustained the damage during transit.

Art. 1735 of the Civil Code provides: In all cases other than those mentioned
in Nos. 1, 2, 3, 4, and 5 of the preceding article, if the goods are lost,
destroyed or deteriorated, common carriers are presumed to have been at
fault or to have acted negligently, unless they prove that they observed
extraordinary diligence as required in Article 1733.

the shipment delivered to the consignee sustained water damage. We agree


with the findings of the CA that petitioner failed to overturn this presumption
Ropa Arienza- Arpilleda

UCPB General Insurance vs. Aboitiz


Feb. 10, 2009

FACTS:

On June 18, 1991, three (3) units of waste water treatment plant with accessories
were purchased by San Miguel Corporation (SMC) from Super Max Engineering
Enterprises, Co., Ltd. of Taipei, Taiwan. The goods came from Charleston, U.S.A. and
arrived at the port of Manila on board MV "SCANDUTCH STAR". The same were then
transported to Cebu on board MV "ABOITIZ SUPERCON II". After its arrival at the port
of Cebu and clearance from the Bureau of Customs, the goods were delivered to and
received by SMC at its plant site on August 2, 1991. It was then discovered that one
electrical motor was damaged. Pursuant to an insurance agreement, plaintiff-appellee
paid SMC the amount representing the value of the damaged unit. In turn, SMC
executed a Subrogation Form in favor of plaintiff-appellee. Consequently, plaintiff-
appellee filed a complaint as subrogee of SMC seeking to recover from defendants the
amount it had paid SMC.

The lower court rendered its assailed Decisiondeclaring DAMCO Intermodal


Systems, Inc., Eagle Express Lines, Inc. and defendant Aboitiz Shipping solidarily liable
to plaintiff-subrogee for the damaged shipment and orders them to pay plaintiff jointly
and severally

The appellate court reversed the decision of the trial court and ruled that UCPB’s
right of action against respondents did not accrue because UCPB failed to file a formal
notice of claim within 24 hours from (SMC’s) receipt of the damaged merchandise as
required under Art. 366 of the Code of Commerce.

ISSUE:
Whether the petitioner may recover from the defendants the amount it has paid
to SMC even if the claim is filed beyond the 24hour limitation required by law.

RULING:

The law clearly requires that the claim for damage or average must be made
within 24 hours from receipt of the merchandise if, as in this case, damage cannot be
ascertained merely from the outside packaging of the cargo. After the periods
mentioned have elapsed, or the transportation charges have been paid, no claim shall
be admitted against the carrier with regard to the condition in which the goods
transported were delivered.

Art. 366 of the Code of Commerce states:

Art. 366. Within twenty-four hours following the receipt of the merchandise, the
claim against the carrier for damage or average which may be found therein upon
opening the packages, may be made, provided that the indications of the damage or
average which gives rise to the claim cannot be ascertained from the outside part of
such packages, in which case the claim shall be admitted only at the time of receipt.

The fundamental reason or purpose of the requirement to give notice of loss or


damage to the goods is not to relieve the carrier from just liability, but reasonably to
inform it that the shipment has been damaged and that it is charged with liability
therefore, and to give it an opportunity to examine the nature and extent of the injury.
This protects the carrier by affording it an opportunity to make an investigation of a
claim while the matter is still fresh and easily investigated so as to safeguard itself from
false and fraudulent claims.

The court have construed the 24-hour claim requirement as a condition


precedent to the accrual of a right of action against a carrier for loss of, or damage to,
the goods. The shipper or consignee must allege and prove the fulfillment of the
condition. Otherwise, no right of action against the carrier can accrue in favor of the
former.
The shipment in this case was received by SMC on August 2, 1991. However, as
found by the Court of Appeals, the claims were dated October 30, 1991, more than
three (3) months from receipt of the shipment and, at that, even after the extent of the
loss had already been determined by SMC’s surveyor. The claim was, therefore, clearly
filed beyond the 24-hour time frame prescribed by Art. 366 of the Code of Commerce.
Silhouette B. Adobas

PHIL. FIRST INSURANCE vs WALLEM PHILS. SHIPPING


G.R. No. 165647 / 26 March 2009

FACTS:

Anhui Chemicals Import & Export Corporation loaded on board M/S


Offshore Master a shipment consisting of 10,000 bags of sodium sulphate
anhydrous 99 PCT Min. (shipment), complete and in good order for
transportation to and delivery at the port of Manila for consignee, L.G.
Atkimson Import-Export, Inc. (consignee), covered by a Clean Bill of Lading.
The Bill of Lading reflects the gross weight of the total cargo at 500,200
kilograms. The Owner and/or Charterer of M/V Offshore Master is unknown
while the shipper of the shipment is Shanghai Fareast Ship Business
Company. Both are foreign firms doing business in the Philippines, thru its
local ship agent, respondent Wallem Philippines Shipping, Inc. (Wallem).

It is undisputed that the shipment was damaged prior to its receipt by


the insured consignee. The damage to the shipment was documented by the
turn-over survey and Request for Bad Order Survey. The turn-over survey, in
particular, expressly stipulates that 2,426 bags of the shipment were
received by the arrastre operator in damaged condition. With these
documents, petitioner insists that the shipment incurred damage or losses
while still in the care and responsibility of Wallem and before it was turned
over and delivered to the arrastre operator.

The trial court, however, found through the testimony of Mr. Maximino
Velasquez Talens, a cargo surveyor of Oceanica Cargo Marine Surveyors
Corporation, that the losses and damage to the cargo were caused by the
mishandling of the arrastre operator. Specifically, that the torn cargo bags
resulted from the use of steel hooks/spikes in piling the cargo bags to the
pallet board and in pushing the bags by the stevedores of the arrastre
operator to the tug boats then to the ports. The appellate court affirmed the
finding of mishandling in the discharge of cargo and it served as its basis for
exculpating respondents from liability, rationalizing that with the fault of the
arrastre operator in the unloading of the cargo established it should bear sole
liability for the cost of the damaged/lost cargo.

ISSUE:

May the Petitioner, in the exercise of its right of subrogation,recover


from the carrier?
RULING:

Section 3(2) of the COGSA states that among the carriers’ responsibilities are to
properly and carefully load, care for and discharge the goods carried. The bill of lading
covering the subject shipment likewise stipulates that the carrier’s liability for loss or
damage to the goods ceases after its discharge from the vessel. Article 619 of the Code
of Commerce holds a ship captain liable for the cargo from the time it is turned over to
him until its delivery at the port of unloading.

It is settled in maritime law jurisprudence that cargoes while being unloaded


generally remain under the custody of the carrier. In the instant case, the damage or
losses were incurred during the discharge of the shipment while under the supervision
of the carrier. Consequently, the carrier is liable for the damage or losses caused to the
shipment. As the cost of the actual damage to the subject shipment has long been
settled, the trial court’s finding of actual damages in the amount of P397,879.69 has to
be sustained.
Kenneth B. Minglana

EASTERN SHIPPING LINES vs PRUDENTIAL GUARANTEE AND ASSURANCE


599 SCRA 565 (2009)

FACTS:

Cargoes belonging to Nissan were shipped by Eastern Shipping Lines. Upon


arrival to the warehouse of NIssan, it was found out that four of the 56 cases were
damaged. As insurer, Prudential Guarantee paid Nissan for the loss in the total amount
of P1,047,298.34. Respondent then sued Petitioner for the collection of what it has paid.
To prove his right of subrogation, Prudential presented as evidence a MARINE CARGO
RISK NOTE and a SUBROGATION RECEIPT. Eastern shipping primarily contends that
such documents are not sufficient to establish the right of subrogation. What is
necessary is the presentation of the MARINE INSURANCE POLICY ITSELF which
covers the goods which were damaged.

ISSUE:

Is the presentation of a marine cargo risk note and a subrogation receipt


sufficient to establish the right of subrogation?

RULING:

The Marine Risk Note relied upon by respondent as the basis for its claim
for subrogation is insufficient to prove said claim. A marine risk note is not an
insurance policy. It is only an acknowledgement confirming the specific shipment
covered by the marine policy, the evaluation of the cargo and the chargeable premium.
It is the marine insurance policy which is constitutive of the insurer-insured relationship
from which Prudential can draw its right of subrogation. Moreover, the contract of
marine insurance must also be presented in evidence to indicate the extent of coverage.
Failure to present the original contract of insurance is fatal to the claim of subrogation
by Prudential Guarantee. However, this rule admits of certain exceptions such as when:

a. The loss of the cargo is undoubtedly occurred on board the carrier’s


vessel
b. The existence of the Marine Insurance Policy is admitted by the party
to whom the claim for subrogation is made against

Sundae June A. Jugao

KEPPEL CEBU SHIPYARD, INC. vs.


PIONEER INSURANCE AND SURETY CORP.
601 SCRA 96

FACTS:

Keppel Cebu Shipyard, Inc. (KCSI) and WG&A Jebsens


Shipmanagement, Inc. (WG&A) executed a Shiprepair Agreement wherein
KCSI would renovate and reconstruct WG&A’s M/V "Superferry 3" using its
dry docking facilities pursuant to its restrictive safety and security rules and
regulations. Prior to the execution of the Shiprepair Agreement, "Superferry
3" was already insured by WG&A with Pioneer Insurance and Surety Corp.
(Pioneer) for US$8,472,581.78. On February 8, 2000, in the course of its
repair, M/V "Superferry 3" was gutted by fire. Claiming that the extent of the
damage was pervasive, WG&A declared the vessel’s damage as a "total
constructive loss" and, hence, filed an insurance claim with Pioneer. Pioneer
paid the insurance claim of WG&A in the amount of US$8,472,581.78. WG&A,
in turn, executed a Loss and Subrogation Receipt in favor of Pioneer. Armed
with the subrogation receipt, Pioneer tried to collect from KCSI, but the latter
denied any responsibility for the loss of the subject vessel. As KCSI
continuously refused to pay despite repeated demands, Pioneer, on August
7, 2000, filed a Request for Arbitration before the Construction Industry
Arbitration Commission (CIAC). Pioneer claims that they are the real party in
interest since it has been subrogated to the claim of its assured. Also they
claim that Keppel is clearly liable for the loss of M/V Superferry 3, since the
immediate cause of the fire was the hot work done by Keppel's employee.
Keppel averred that the Claimant is not a real party in interest and has no
standing because it has not been subrogated to the Vessel Owner and the
insurance policies on which the Claimant bases its right of subrogation were
not validly obtained.

ISSUES:

1. Whether or not constructive total loss occurred and WG&A's abandonment


of the ship was proper.
2. Whether or not subrogation is proper. If proper, to what extent can
subrogation be made?

RULING:

1. In marine insurance, a constructive total loss occurs under any of the


conditions set forth in Section 139 of the Insurance Code, which
provides— Sec. 139. A person insured by a contract of marine
insurance may abandon the thing insured, or any particular portion
hereof separately valued by the policy, or otherwise separately
insured, and recover for a total loss thereof, when the cause of the loss
is a peril insured against:

(a) If more than three-fourths thereof in value is actually lost, or would


have to be expended to recover it from the peril;

(b) If it is injured to such an extent as to reduce its value more than


three-fourths;

It cannot be denied that M/V "Superferry 3" suffered widespread


damage from the fire that occurred, a covered peril under the marine
insurance policies obtained by WG&A from Pioneer. The estimates
given by the three disinterested and qualified shipyards show that the
damage to the ship would exceed P270,000,000.00, or ¾ of the total
value of the policies – P360,000,000.00. These estimates constituted
credible and acceptable proof of the extent of the damage sustained
by the vessel. Also, the CA held that Section 139 of the Insurance
Code is merely permissive on account of the word “may” in the
provision. This is incorrect. Properly considered, the word “may” in
the provision is intended to grant the insured (WG&A) the option or
discretion to choose the abandonment of the thing insured, or any
particular portion thereof separately valued by the policy, or otherwise
separately insured, and recover for a total loss when the cause of the
loss is a peril insured against. This option or discretion is expressed as
a right in Section 131 of the same Code, to wit: Sec. 131. A
constructive total loss is one which gives to a person insured a right to
abandon under Section one hundred thirty-nine. Thus, WG&A
abandonment of the ship was proper.

2. Subrogation is proper. Subrogation is the substitution of one person by


another with reference to a lawful claim or right, so that he who is
substituted succeeds to the rights of the other in relation to a debt or
claim, including its remedies or securities. The principle covers a
situation wherein an insurer has paid a loss under an insurance policy
is entitled to all the rights and remedies belonging to the insured
against a third party with respect to any loss covered by the policy. It
contemplates full substitution such that it places the party subrogated
in the shoes of the creditor, and he may use all means that the creditor
could employ to enforce payment. The court have held that payment
by the insurer to the insured operates as an equitable assignment to
the insurer of all the remedies that the insured may have against the
third party whose negligence or wrongful act caused the loss. The
right of subrogation is not dependent upon, nor does it grow out of,
any privity of contract. It accrues simply upon payment by the
insurance company of the insurance claim. The doctrine of
subrogation has its roots in equity. It is designed to promote and to
accomplish justice; and is the mode that equity adopts to compel the
ultimate payment of a debt by one who, in justice, equity, and good
conscience, ought to pay. To allow KCSI to limit its liability to only
P50,000,000.00, notwithstanding the fact that there was a constructive
total loss in the amount of P360,000,000.00, would sanction the
exercise of a degree of diligence short of what is ordinarily required. It
would not be difficult for a negligent party to escape liability by the
simple expedient of paying an amount very much lower than the actual
damage or loss sustained by the other. Nevertheless, the court concur
with the position of KCSI that the salvage value of the damaged M/V
"Superferry 3" should be taken into account in the grant of any award.
Not considering this salvage value in the award would amount to
unjust enrichment on the part of Pioneer.

Junyvil B. Tumbaga

ASIAN TERMINALS, INC. vs. DAEHAN FIRE


AND MARINE INSURANCE CO., LTD.,
G.R. No. 171194, February 4, 2010.

FACTS:

Doosan Corporation shipped twenty-six (26) boxes of printed aluminum sheets


on board the vessel Heung-A Dragon owned by Dongnama Shipping Co., Ltd. and
consigned to Access International. Doosan insured the subject shipment with
respondent Daehan Fire and Marine Insurance Co., Ltd. under an “all-risk” marine cargo
insurance policy, payable to its settling agent in the Philippines, the Smith Bell & Co.,
Inc. The vessel arrived in Manila and the containerized van was discharged and
unloaded in apparent good condition, as no survey and exceptions were noted in the
Equipment Interchange Receipt (EIR) issued by petitioner. The container van was
stored in the Container Yard of the Port. Access International requested from petitioner
and the licensed Customs Broker Lazo a joint survey of the shipment at the place of
storage in the Container Yard, but no such inspection was conducted. Lazo withdrew,
and petitioner released the shipment and delivered it to Access International’s
warehouse in. While the shipment was at Access International’s warehouse, the latter,
together with its surveyor, Lloyd’s Agency, conducted an inspection and noted that only
twelve (12) boxes were accounted for, while fourteen (14) boxes were missing. Access
International thus filed a claim against petitioner and Lazo for the missing shipment. For
failure to collect its claim, Access International sought indemnification from respondent
Daehan. Respondent paid the amount of the claim and Access International accordingly
executed a Subrogation Receipt in favor of the former. Respondent, represented by
Smith Bell, instituted the present case before the RTC. Respondent alleged that the
losses, shortages and short deliveries sustained by the shipment were caused by the
joint fault and negligence of petitioner and Lazo. Petitioner denied liability, claiming that
it exercised due diligence in handling and storing the subject container van.

ISSUE:

Whether or not respondent, as insurer, was subrogated to the rights of


the consignee.

RULING:

Respondent, as insurer, was subrogated to the rights of the consignee,


pursuant to the subrogation receipt executed by the latter in favor of the
former. Subrogation is the substitution of one person by another with
reference to a lawful claim or right, so that he who is substituted succeeds to
the rights of the other in relation to a debt or claim, including its remedies or
securities. The principle covers a situation wherein an insurer has paid a loss
under an insurance policy is entitled to all the rights and remedies belonging
to the insured against a third party with respect to any loss covered by the
policy. The payment by the insurer to the insured operates as an equitable
assignment to the insurer of all the remedies that the insured may have
against the third party whose negligence or wrongful act caused the loss.

William Z. Radaza

ASIAN TERMINALS, INC. vs. MALAYAN INSURANCE CO., INC.


647 SCRA 111; Apr. 4, 2011

FACTS:

Respondent Malayan Insurance was the insurer of a shipment consisting of


plastic bags of soda ash dense covered by a Marine Insurance Policy. When the
shipment arrived in Manila, the stevedorers (persons employed to unload cargoes) of
petitioner Asian Terminals unloaded the shipment. The consignee found out that some
of the cargoes were in bad condition so the insurer paid the value lost or damaged to
the consignee. Malayan Insurance, as subrogee of the consignee, filed a complaint for
damages against Asian Terminals for causing the loss due to negligence in unloading
the cargo.

Asian Terminals argued that Malayan Insurance has no cause of action because
it failed to present the insurance contract or policy covering the subject shipment. It also
alleged that the Subrogation Receipt presented by Malayan Insurance is not sufficient to
prove that the subject shipment was insured and that the insurer was validly subrogated
to the rights of the consignee.

Malayan Insurance, on the other hand, claimed that under prevailing


jurisprudence, the presentation of the insurance policy is not indispensible.

ISSUE:

Whether or not the presentation of the insurance contract or policy is


indispensible before the insurer may recover from the carrier.

RULING:

The presentation in evidence of the marine insurance policy is not indispensable


before the insurer may recover from the common carrier the insured value of the lost
cargo in the exercise of its subrogatory right. The subrogation receipt, by itself, is
sufficient to establish not only the relationship of the insurer and the assured shipper of
the lost cargo, but also the amount paid to settle the insurance claim. The right of
subrogation accrues simply upon payment by the insurer company of the insurance
claim.

Aurora Luanne R. Cembrano

DEVELOPMENT INSURANCE CORPORATION vs. IAC


143 SCRA 62 / July 16, 1986

FACTS:
A fire occurred in a building owned by private respondent, destroying a portion of the 7th
floor thereof. The building is insured at P2,500,000.00 under an open policy and is still under
construction when the fire occurred. Private respondent now sues petitioner for recovery of
damages based on the insurance contract between them.

ISSUE:

How much may private respondent recover as indemnity from petitioner?

RULING:

Under Sec. 60 of the Insurance Code, an open policy is one in which the value of the
thing insured is not agreed upon but is left to be ascertained in case of loss. Meaning, the actual
loss will represent the total indemnity due the insured from the insurer except only that it shall
not exceed the face value of the policy.

Applying the open policy clause, the Supreme Court ruled that private respondent is
entitled to the payment of indemnity under said contract in the total amount of P508,867.00, the
actual value of the loss incurred.

Christine Mae P. Navarra


ARMANDO GEAGONIA vs. COURT OF APPEALS
and COUNTRY BANKERS INSURANCE CORPORATION
G.R. No. 114427 February 6, 1995

FACTS:

Petitioner Geagoniais the owner of Norman's Mart located in the public


market of San Francisco, Agusandel Sur. He obtained from the private
respondent a fire insurance policy for P100,000.00 for the period of
December 22, 1989 to December 22, 1990 and covered the following:
"Stock-in-trade consisting principally of dry goods such as RTW's for men and
women wear and other usual to assured's business." On May 27, 1990, fire of
accidental origin broke out at the public market of San Francisco, Agusandel
Sur. The petitioner's insured stock-in-trade was completely destroyed
prompting him to file with the private respondent a claim under the policy.
Country Bankers denied the claim because it found that at the time of the
loss the petitioner's stocks-in-trade were likewise covered by fire insurance
policies issued by the Cebu Branch of the Philippines First Insurance Co., Inc.
(hereinafter PFIC). The basis of the private respondent's denial was the
petitioner's alleged violation of Condition 3 of the policy which contains that:

“The insured shall give notice to the Company of any insurance or


insurances already affected, or which may subsequently be effected…
xxx and unless such notice be given and the particulars of such
insurance or insurances be stated therein or endorsed in this policy
pursuant to Section 50 of the Insurance Code… xxx all benefits under
this policy shall be deemed forfeited, provided however, that this
condition shall not apply when the total insurance or insurances in
force at the time of the loss or damage is not more than P200,000.00.

The petitioner then filed a complaint against the private respondent


with the Insurance Commission for the recovery of P100,000.00 under fire
insurance policy and for attorney's fees and costs of litigation. He admitted
that at the time he obtained the private respondent's fire insurance policy he
knew that the two policies issued by the PFIC were already in existence;
however, he had no knowledge of the provision in the policy requiring him to
inform it of the prior policies; and had it been mentioned, he would not have
withheld such information. The Insurance Commission found that the
petitioner did not violate Condition 3 as he had no knowledge of the
existence of the two fire insurance policies obtained from the PFIC; that it
was Cebu Tesing Textiles which procured the PFIC policies without informing
him or securing his consent; and that Cebu Tesing Textile, as his creditor, had
insurable interest on the stocks.

Private respondents appealed to the Court of Appeals which reverses


the same because it found that the petitioner knew of the existence of the
two other policies issued by the PFIC. His motion to reconsider the adverse
decision having been denied, Geagonia filed the petition for review on
certiorari.

ISSUES:

1. Whether the non-disclosure of other insurance policies violate


condition 3 of the policy, so as to deny Geagonia from recovering on
the policy
2. Whether the violation of Condition 3 of the policy renders the policy
void

RULING:

1. Condition 3 of the private respondent's Policy No. F-14622 is a


condition which is not proscribed by law. Its incorporation in the policy
is allowed by Section 75 of the Insurance Code which provides that "[a]
policy may declare that a violation of specified provisions thereof shall
avoid it, otherwise the breach of an immaterial provision does not
avoid the policy." Such a condition is a provision which invariably
appears in fire insurance policies and is intended to prevent an
increase in the moral hazard. It is commonly known as the additional or
"other insurance" clause and has been upheld as valid and as a
warranty that no other insurance exists. Its violation would thus avoid
the policy. However, in order to constitute a violation, the other
insurance must be upon same subject matter, the same interest
therein, and the same risk.

As to a mortgaged property, the mortgagor and the mortgagee


have each an independent insurable interest therein and both interests
may be one policy, or each may take out a separate policy covering his
interest, either at the same or at separate times. The mortgagor's
insurable interest covers the full value of the mortgaged property,
even though the mortgage debt is equivalent to the full value of the
property. The mortgagee's insurable interest is to the extent of the
debt, since the property is relied upon as security thereof, and in
insuring he is not insuring the property but his interest or lien thereon.
His insurable interest is prima facie the value mortgaged and extends
only to the amount of the debt, not exceeding the value of the
mortgaged property. Thus, separate insurances covering different
insurable interests may be obtained by the mortgagor and the
mortgagee.

A double insurance exists where the same person is insured by


several insurers separately in respect of the same subject and interest.
As earlier stated, the insurable interests of a mortgagor and a
mortgagee on the mortgaged property are distinct and separate. Since
the two policies of the PFIC do not cover the same interest as that
covered by the policy of the private respondent, no double insurance
exists. The non-disclosure then of the former policies was not fatal to
the petitioner's right to recover on the private respondent's policy.

2. Condition 3 in the private respondent's policy No. F-14622 does not


absolutely declare void any violation thereof. It expressly provides that
the condition "shall not apply when the total insurance or insurances in
force at the time of the loss or damage is not more than P200,000.00.

It is a cardinal rule on insurance that a policy or insurance


contract is to be interpreted liberally in favour of the insured and
strictly against the company, the reason being, undoubtedly, to afford
the greatest protection which the insured was endeavouring to secure
when he applied for insurance. It is also a cardinal principle of law that
forfeitures are not favoured and that any construction which would
result in the forfeiture of the policy benefits for the person claiming
thereunder, will be avoided, if it is possible to construe the policy in a
manner which would permit recovery, as, for example, by finding a
waiver for such forfeiture.

Albert G. Cong

SPOUSES CHA vs. COURT OF APPEALS


277 SCRA 690 (1997)

FACTS:

Spouses Nilo Cha and Stella Uy-Cha (spouses) and CKS Development
Corporation (CKS) entered a 1 year lease contract with a stipulation that the lessee, the
spouses, must not insure against fire the chattels at any space in the leased premises
without the written consent and approval of the lessor, CKS. But the spouses insured
against loss by fire their merchandise inside the leased premises for P500,000.00 with
the United Insurance Co. (United) without CKS’ written consent.

On the day the lease contract was to expire, fire broke out inside the leased
premises. CKS learned that the spouses procured insurance and wrote to United to
have the proceeds be paid directly to them but United refused. CKS filed suit against
the spouses and United. The RTC ordered United to pay CKS the amount of
P335,063.11 and the spouses to pay for exemplary damages, attorney’s fees and costs
of suit.

ISSUE:

Does CKS have insurable interest due to the spouses’ violation to the stipulation
in the lease contract?

RULING:

No. A non-life insurance policy such as the fire insurance policy taken by the spouses
over their merchandise is primarily a contract of indemnity. Sec. 18 of the Insurance
Code of the Philippines states that no contract or policy of insurance on property shall
be enforceable except for the benefit of some person having an insurable interest in the
property insured. The basis of such requirement of insurable interest in property
insured is based on sound public policy: to prevent a person from taking out an
insurance policy on property upon which he has no insurable interest and collecting the
proceeds of said policy in case of loss of the property.

The automatic assignment of the policy to CKS under the provision of the lease contract
previously quoted is void for being contrary to law and/or public policy. The proceeds of
the fire insurance policy thus rightfully belong to the spouses. Their liability to CKS for
violating their lease contract is a separate and distinct issue which the Supreme Court
did not resolve in this case.
Rogaciano A. Quico III

GREAT PACIFIC LIFE ASSURANCE CORP. vs.


COURT OF APPEALS AND MEDARDA V. LEUTERIO
316 SCRA 677 / Oct. 13, 1999

FACTS:

This is a petition for review under Rule 45 of the Rules of Court, assailing the
decision and resolution of the Court of Appeals dated May 17, 1994 and January 4,
1994, respectively, in CA G.R. CV No. 18341. The appellate court affirmed in toto the
judgment of the Regional Trial Court of Misamis Oriental in an insurance claim filed by
private respondent against Great Pacific Life Assurance Co.

The Supreme Court found the petition not meritorious. Contrary to petitioner's
allegations, there was no sufficient proof that the insured had suffered from
hypertension. Aside from the statement of the insured's widow who was not even sure if
the medicines taken by Dr. Leuterio were for hypertension, the petitioner had not proven
nor produced any witness who could attest to Dr. Leuterio's medical history. Clearly, it
had failed to establish that there was concealment made by the insured, hence it cannot
refuse payment of the claim.

ISSUES:

Whether Dr. Leuterio concealed that he had hypertension, which would vitiate the
insurance contract?

RULING:

NO. Contrary to appellant's allegations, there was no sufficient proof that the
insured had suffered from hypertension. Aside from the statement of the insured's
widow who was not even sure if the medicines taken by Dr. Leuterio were for
hypertension, the appellant had not proven nor produced any witness who could attest
to Dr. Leuterio's medical history . . . Appellant insurance company had failed to establish
that there was concealment made by the insured, hence, it cannot refuse payment of
the claim." The fraudulent intent on the part of the insured must be established to entitle
the insurer to rescind the contract. Misrepresentation as a defense of the insurer to
avoid liability is an affirmative defense and the duty to establish such defense by
satisfactory and convincing evidence rests upon the insurer. In the case at bar, the
petitioner failed to clearly and satisfactorily establish its defense, and is therefore liable
to pay the proceeds of the insurance.
Mary Christine Anthonette M. Salise-Punzalan

PHILAMCARE HEALTH SYSTEMS, INC.


vs. COURT OF APPEALS and JULITA TRINOS
G.R. No. 125678 March 18, 2002

FACTS:

ErnaniTrinos, deceased husband of JulitaTrinos, applied for a health


care coverage with Philamcare Health Systems, Inc. In thestandard
application form, he answered “NO” to the following question:

Have you or any of your family members ever consulted or


been treated for high blood pressure, heart trouble, diabetes, cancer,
liver disease, asthma or peptic ulcer? (If Yes, give details).

Coverage of the health care agreement (HCA):

•approved for a period of one year, Renewed 3 times yearly: March


1, 1988 - March 1, 1990; March 1, 1990 – June 1, 1990. The amount of
coverage was increased to a maximum sum of P75,000.00 per
disability.Ernani’s entitlement under HCA:

•hospitalization benefits, whether ordinary or emergency, listed therein


•out-patient benefits" such as annual physical examinations, preventive
health care and other out-patient services.Ernaniwas subsequently confined.
HISTORY (everything happened within the period of coverage):1.Ernani
suff ered a heart attack and was confi ned at the Manila Medical
Center (MMC) for one month beginning March 9, 1990.2. Julita tried
to claim the benefi ts under the health care
agreement.3.Philamdenied her claim saying that the Health Care
Agreement was void. there was a concealment regarding Ernani’s
medical history.

Doctors at the MMC allegedly discovered at the time of Ernani’s


confi nement that he was hypertensive, diabetic and asthmatic,
contrary to his answer in the application form.4.Julita paid the
hospitalization expenses herself, amounting to about
P76,000.005 . E r n a n i w a s d i s c h a r g e d a t M M C . H e w a s a t t e n d e d
by a physical therapist at home. Again he was admitted at the
C h i n e s e G e n e r a l H o s p i t a l . Julita brought her husband home again
due to fi nancial diffi culties. In the morning of April 13, 1990, Ernani
had fever and was feeling very weak . Julita was constrained to bring him
back to the Chinese General Hospital where he died on the same day.On July
24, 1990, respondent instituted with the Regional Trial Court of Manila,
Branch 44, an action for damages against Philamandits president, Dr. Benito
Reverente, She asked for reimbursement of her expenses plus moral
damages and attorney’s fees. After trial,the lower court ruled against Philam,
ordered:

1. Defendants to pay and reimburse the medical and hospital


coverage of the late ErnaniTrinos in the amount of P76,000.00 plus
interest, until the amount is fully paid to plaintiff who paid the same;
2. Defendants to pay the reduced amount of moral damages of P10,000.00
to plaintiff;
3. Defendants to pay the reduced amount ofP10,000.00 as exemplary
damages to plaintiff;
4. Defendants to pay attorney’s fees of P20,000.00, plus costs of suit.CA:
affirmed the decision of the trial court but deleted all awards for damages
and absolved petitioner Reverente. Denied MR.

ISSUES:

1.Whether health care agreements are considered insurance


contracts.
2.Whether there was concealment of material facts on the part of
Ernani that rendered the HCA void by virtue of the "Invalidation of
agreement" contained in the contract.
3.Suppose there was concealment, what are the steps Philam should
have done?

RULING:

1.YES, it is an insurance contract.

Section 2 (1) of the Insurance Code defi nes a contract of insurance


as an agreement whereby one undertakes for a consideration to
indemnify another against loss, damage or liability arising from an unknown
or contingent event. An insurance contract exists where the following
elements concur:
(1) The insured has an insurable interest;
(2) The insured is subject to a risk of loss by the happening of the
designated peril;
(3) The insurer assumes the risk;
(4) Such assumption of risk is part of a general scheme to
distribute actual losses among a large group of persons bearing
a similar risk; and
(5) In consideration of the insurer’s promise, the insured pays a
premium.Section 3 of the Insurance Code states that any
contingent or unknown event, whether past or future, which
may damnify a person having an insurable interest against him,
may be insured against. Every person has an insurable interest
in the life and health of himself. Section 10 provides:Every person
has an insurable interest in the life and health:(1) of himself, of his
spouse and of his children;(2) of any person on whom he depends
wholly or in part for education or support, or in whom he has a
pecuniary interest.
Ropa Arienza- Arpilleda

BLUE CROSS HEALTH CARE, INC. vs. NEOMI AND DANILO OLIVARES
Feb. 12, 2008

FACTS:

Respondent Neomi T. Olivares applied for a health care program with petitioner
Blue Cross Health Care, Inc., a health maintenance firm. She paid the amount
of P11,117 and availed of the additional service of limitless consultations for an
additional amount of P1,000. She paid these amounts in full on October 17, 2002. The
application was approved on October 22, 2002. In the health care agreement, ailments
due to “pre-existing conditions” were excluded from the coverage.

Barely 38 days from the effectivity of her health insurance, respondent Neomi
suffered a stroke. During her confinement, she underwent several laboratory tests and
incurred hospital expenses amounting to P34,217.20. She requested from the
representative of petitioner a letter of authorization in order to settle her medical bills.
But petitioner refused to issue the letter and suspended payment pending the
submission of a certification from her attending physician that the stroke she suffered
was not caused by a pre-existing condition.

ISSUE:

Whether or not respondent has the burden of proof in showing that her stroke
was caused by a pre-existing condition and therefore was excluded from the coverage
of the health care agreement.
RULING:

It was the burden of petitioner to prove that the stroke suffered by respondent
Neomi was excluded from the coverage of the health care program for being caused by
a pre-existing condition. Petitioner never presented any evidence to prove that
respondent Neomi's stroke was due to a pre-existing condition. It merely speculated that
Dr. Saniel's report would be adverse to Neomi, based on her invocation of the doctor-
patient privilege.

Furthermore, limitations of liability on the part of the insurer or health care


provider must be construed in such a way as to preclude it from evading its
obligations. Since petitioner had the burden of proving exception to liability, it should
have made its own assessment of whether respondent Neomi had a pre-existing
condition when it failed to obtain the attending physician's report. It could not just
passively wait for Dr. Saniel's report to bail it out. The mere reliance on a disputable
presumption does not meet the strict standard required under our jurisprudence.

Silhouette B. Adobas

NATIONAL POWER CORP vs. COURT OF APPEALS


145 SCRA 533 / 14 November 1986

FACTS:

The National Power Corporation (NPC) entered into a contract with the Far Eastern
Electric, Inc. (FFEI) on December 26, 1962 for the erection of the Angat Balintawak 115-KW-3-
Phase transmission lines for the Angat Hydroelectric Project. FEEI agreed to complete the work
within 120 days from the signing of the contract, otherwise it would pay NPC P200.00 per
calendar day as liquidated damages, while NPC agreed to pay the sum of P97,829.00 as
consideration. On the other hand, Philippine American General Insurance Co., Inc. (Philamgen)
issued a surety bond in the amount of P30,672.00 for the faithful performance of the undertaking
by FEEI, as required.

FEEI started construction on December 26, 1962 but on May 30, 1963, both
FEEI and Philamgen wrote NPC requesting the assistance of the latter to complete the
project due to unavailability of the equipment of FEEI. The work was abandoned on
June 26, 1963, leaving the construction unfinished. On July 19, 1963, in a joint letter,
Philamgen and FEEI informed NPC that FEEI was giving up the construction due to
financial difficulties. On the same date, NPC wrote Philamgen informing it of the
withdrawal of FEEI from the work and formally holding both FEEI and Philamgen liable
for the cost of the work to be completed as of July 20, 1962 plus damages.

ISSUE:
Whether or not National Power Corporation complied with the notice requirement
as a condition in order to hold the surety liable under the bond.

RULING:

As correctly assessed by the trial court, the evidence on record shows that as early as
May 30, 1963, Philamgen was duly informed of the failure of its principal to comply with its
undertaking. In fact, said notice of failure was also signed by its Assistant Vice President. On
July 19, 1963, when FEEI informed NPC that it was abandoning the construction job, the latter
forthwith informed Philamgen of the fact on the same date. Moreover, on August 1, 1963, the
fact that Philamgen was seasonably notified, was even bolstered by its request from NPC for
information of the percentage completed by the bond principal prior to the relinquishment of the
job to the latter and the reason for said relinquishment. (Record on Appeal, pp. 193-195). The 30-
day notice adverted to in the surety bond applies to the completion of the work by the contractor.
This completion by the contractor never materialized.

The surety bond must be read in its entirety and together with the contract between NPC
and the contractors. The provisions must be construed together to arrive at their true meaning.
Certain stipulations cannot be segregated and then made to control.

Furthermore, it is well settled that contracts of insurance are to be construed liberally in


favor of the insured and strictly against the insurer. Thus ambiguity in the words of an insurance
contract should be interpreted in favor of its beneficiary.

Kenneth B. Minglana

PRUDENTIAL GUARANTY AND ASSURANCE vs EQUINOX LAND CORPORATION


533 SCRA 257 (2007)

FACTS:

Equinox contracted with J’Marc Construction to construct 5 additional floors to its


building. J’Marc submitted to Equinox 2 bonds namely: A surety bond to guarantee the
unliquidated portion of the advance payment and a performance bond to guarantee
J’Marc’s faithful performance of its obligation all issued by Prudential Guaranty, herein
petitioner.

J’Marc did not adhere to the terms of the construction contract to the point that it
has miserably failed to comply its obligations with Equinox. Consequently, Equinox filed
a case for sum of money with damages against J’Marc and also prayed that Prudential
guaranty be ordered to pay its liability under the bonds. Prudential Guaranty contends
that it is not bound by the construction contracts between Equinox and Jmarc and it
cannot be held solidarily liable with JMarc under its bonds.

ISSUE:

What is the nature of the liability of a Surety?

RULING:
It is not disputed that Prudential entered into a suretyship contract with J’Marc. Section
175 of the Insurance Code defines a suretyship as "a contract or agreement whereby a party,
called the suretyship, guarantees the performance by another party, called the principal or
obligor, of an obligation or undertaking in favor of a third party, called the obligee. It includes
official recognizances, stipulations, bonds, or undertakings issued under Act 536, as amended."
Corollarily, Article 2047 of the Civil Code provides that suretyship arises upon
the solidary binding of a person deemed the surety with the principal debtor for the purpose of
fulfilling an obligation. While a contract of surety is secondary only to a valid principal
obligation, the surety’s liability to the creditor is said to be direct, primary, and absolute. In other
words, the surety is directly and equally bound with the principal. Thus, Prudential is barred
from disclaiming that its liability with J’Marc is solidary.

Sundae June A. Jugao

INTRA-STRATA ASSURANCE CORPORATION and PHILIPPINE HOME


ASSURANCE CORPORATION vs. REPUBLIC OF THE PHILIPPINES
557 SCRA 363

FACTS:

Grand Textile is a local manufacturing corporation which imported from different


countries various articles. Subsequent to the importation, these articles were transferred
to Customs Bonded Warehouse No. 462. As computed by the Bureau of Customs, the
customs duties, internal revenue taxes, and other charges due on the importations
amounted to P2,363,147.00. To secure the payment of these obligations pursuant to the
Tariff and Customs Code (Code), Intra-Strata and PhilHome each issued general
warehousing bonds in favor of the Bureau of Customs. These bonds commonly provide
that the goods shall be withdrawn from the bonded warehouse "on payment of the legal
customs duties, internal revenue, and other charges to which they shall then be
subject.” Grand Textile withdrew the articles without payment of the taxes, customs
duties, and charges due. The Bureau of Customs demanded payment of the amounts
due from Grand Textile as importer, and from Intra-Strata and PhilHome as sureties. All
three failed to pay. The government filed a collection suit against the parties with the
RTC of Manila. RTC held Grand Textile (as importer) and the petitioners (as sureties)
liable for the taxes, duties, and charges due on the imported articles, which is also
affirmed by the CA.

ISSUE:

Whether or not the withdrawal of the imported articles, without notice to the
petitioners as sureties, released them from any liability.

RULING:

Petitioners are not released from liability. Section 175 of the Insurance Code
defines a contract of suretyship as an agreement whereby a party called the surety
guarantees the performance by another party called the principal or obligor of an
obligation or undertaking in favor of another party called the obligee, and includes
among its various species bonds such as those issued pursuant to Section 1904 of the
Code. The liability of the surety is joint and several but limited to the amount of the bond,
and its terms are determined strictly by the terms of the contract of suretyship in relation
to the principal contract between the obligor and the obligee. A surety is released from
its obligation when there is a material alteration of the contract in connection with which
the bond is given, such as a change which imposes a new obligation on the promising
party, or which takes away some obligation already imposed, or one which changes the
legal effect of the original contract and not merely its form. A surety, however, is not
released by a change in the contract which does not have the effect of making its
obligation more onerous. However, the court finds under the facts of this case no
significant or material alteration in the principal contract between the government and
the importer, or in the obligation that the petitioners assumed as sureties. The surety
does not, by reason of the surety agreement, earn the right to intervene in the principal
creditor-debtor relationship; its role becomes alive only upon the debtor’s default, at
which time it can be directly held liable by the creditor for payment as a solidary obligor.
A surety contract is made principally for the benefit of the creditor-obligee and this is
ensured by the solidary nature of the sureties’ undertaking. Under these terms, the
surety is not entitled as a rule to a separate notice of default, nor to the benefit of
excussion, and may be sued separately or together with the principal debtor.

Junyvil B. Tumbaga

COUNTRY BANKERS INSURANCE CO. vs. LAGMAN


G.R. No. 165487 July 13, 2011

FACTS:

Country Bankers Insurance Corporation issued Warehouse Bonds by


which Nelson Santos was the bond principal, Lagman was the surety and the
Republic of the Philippines, through the NFA was the obligee. The
said bonds were used by Nelson as a requirement for his application for Warehouse
business. In consideration of these issuances, corresponding Indemnity Agreements
were executed by Santos, as bond principal, together with Ban Lee Lim, Santos,
Reguine and Lagman, as co-signors. The latter bound
themselves jointly and severally liable to Country Bankers for any damages which it
may sustain as a consequence of the said bond. Santos then secured a loan using his
warehouse receipts as collateral. When the loan matured, Santos defaulted in his
payment. By virtue of the surety bonds, Country Bankers was compelled to pay
P1,166,750.37. Consequently, Country Bankers filed a complaint for a sum of money.
The bond principals, Santos and Ban
LeeLim, were not served with summons because they could no longer be found. The
case was eventually dismissed against them without prejudice. The trial court rendered
judgment declaring Reguine and Lagman jointly and severally liable to pay Country
Bankers. Lagman filed an appeal to the Court of Appeals. He insisted that the lifetime of
the 1989 Bonds, as well as the corresponding Indemnity Agreements was only 12
months. The CA reversed the decision of the trial court.

ISSUE:

Whether or not the warehouse bonds were effective only for one year.

RULING:

No.The official receipts in question serve as proof of payment of thepre


mium for
one year on each surety bond. It does not, however, automatically mean that
the surety bond is effective for only one (1) year. In fact, the effectivity of the
bond is not wholly dependent on the payment of premium. Section 177 of
the Insurance Code expresses: Sec. 177. The surety is entitled to payment of
the premium as soon as the contract of suretyship or bond is perfected and
delivered to the obligor. No contract of suretyship or bonding shall be valid
and binding unless and until the premium therefor has been paid,
except where the obligee has accepted the
bond,in which case the bond becomes valid and enforceable
irrespective of whether or not the premium has been paid by the
obligor to the surety. Provided, that if the contract of suretyship or bond is
not accepted by, or filed with the obligee, the surety shall collect only
reasonable amount, not exceeding fifty per centum of the premium due
thereon as service fee plus the cost of stamps or other taxes imposed for the
issuance of the contract or bond: Provided, however, That if the non-
acceptance of the bond be due to the fault or negligence of the surety, no
such service fee, stamps or taxes shall be collected.

William Z. Radaza

THE MANILA INSURANCE COMPANY, INC. vs. SPS. AMURAO


G.R. No. 179628; Jan. 16, 2013

FACTS:

Respondent spouses, Roberto and Aida Amurao, entered into a Construction


Contract Agreement (CCA) with Aegean Construction and Development Corporation for
the construction of a commercial building. To guarantee full and faithful compliance with
the terms and conditions of the CCA, Aegean posted performance bond secured by
petitioner Manila Insurance Company. Due to failure of Aegean to complete the project,
respondent spouses filed with the RTC a complaint to collect on the performance of the
bond against Manila Insurance.
The surety, Manila Insurance, filed a motion to dismiss on the ground that the
complaint did not state a cause of action and that it failed to implead Aegean, the
principal contractor. It argued that, while a surety is bound solidarily with the obligor, it
does not make the surety a solidary co-debtor. It also contended that a surety is liable
only if the debtor is himself liable. The dispute between Aegean, the contractor, and the
spouses should first be resolved before the Construction Industry Arbitration
Commission.

ISSUE:

Whether or not Manila Insurance, as surety, is directly and solidarily liable with
Aegean, the obligor for failure to perform the Construction Contract Agreement.

RULING:

Manila Insurance is directly and solidarily liable under the suretyship agreement.
A contract of suretyship is defined as an agreement whereby a party, called the surety,
guarantees the performance by another party, called the principal or obligor, of an
obligation or undertaking in favor of a third party, called the obligee. It includes official
recognizances, stipulations, bonds or undertakings issued by any company by virtue of
and under the provisions of Act No. 536, as amended by Act No. 2206.

A surety’s liability is joint and several (or solidary), limited to the amount of the
bond, and determined strictly by the terms of the contract of suretyship in relation to the
principal contract between the obligor and the obligee. It bears stressing, however, that
although the contract of suretyship is secondary to the principal contract, the surety’s
liability to the obligee is nevertheless direct, primary, and absolute.

Aurora Luanne R. Cembrano

IN RE MARIO V. CHANLIONGCO
79 SCRA 364 / Oct. 18, 1977

FACTS:

This refers to the claims for retirement benefits filed by the heirs of the
late ATTY. MARIO V. CHANLIONGCO which includes his retirement benefits.
His heirs include his widow, one legitimate son, and two illegitimate children.

The record shows that the deceased died ab intestato and that he
failed or overlooked to state in his application for membership with the GSIS
the beneficiary or beneficiaries of his retirement benefits, should he die
before retirement.
ISSUE:

How must the retirement benefits be distributed?

RULING:

When a government employee failed to name the beneficiary of his


retirement benefits, the proceeds thereof shall accrue to his estate and will
be distributed among his legal heirs in accordance with the law on intestate
succession, as in the case of a life insurance if no beneficiary is named in the
insurance policy. Hence, the decedents estate is distributed in the proportion
of ½ for the legitimate child, ¼ for the widow and 1/8 each for the two
illegitimate children.

Christine Mae P. Navarra

THE INSULAR LIFE ASSURANCE CO., LTD. vs.


CARPONIA T. EBRADO AND PASCUALA VDA. DE EBRADO
G.R. No. L-44059 October 28, 1977

FACTS:

Buenaventura CristorEbrado was issued by the Insular Life Insurance Co., Ltd.,
Policy No. 009929 on a whole-life plan for P5,882.00 with a rider for Accidental Death
Benefits for the same amount. He designated Carponia T. Ebrado as the revocable
beneficiary of which he referred to her as his wife.

Later, Mr.Ebrado died as a result of an accident when he was hit by a falling


branch of a tree. As the insurance policy was in force, the insurance company stands
liable to pay the coverage. CarponiaEbrado filed with the insurer a claim for the
proceeds of the policy as the designated beneficiary therein, although she admits that
she and the insured were merely living as husband and wife without the benefit of
marriage. PascualaVda. De Ebrado also filed her claim as the widow of the deceased
insured. She asserts that she is the one entitled to the insurance proceeds, and not the
common-law wife.

In doubt as to whom the insurance proceeds shall be paid, the insurer


commenced an action for Interpleader before the CFI of Rizal. The court rendered
judgment declaring among others, Carponia T. Ebrado disqualified from becoming
beneficiary of the insured Mr. Buenaventura and directing the payment of the insurance
proceeds to the estate of the deceased insured. From this judgment, Carponia T.
Ebrado appealed to the Court of Appeals but the Appellate Court certified the case to
the Supreme Court as involving only questions of law.

ISSUES:

1. Whether common-law wife named as beneficiary in the life insurance policy of a


legally married man claim the proceeds thereof in case of death of the latter

2. Whether a conviction for adultery or concubinage is exacted before the


disabilities mentioned in Article 739 may effectuate

RULING:

1. NO. It is quite unfortunate that the Insurance Act or even the new Insurance
Code does not contain any specific provision grossly resolutory of the prime
question at hand. Section 50 of the Insurance Act which provides that “the
insurance shall be applied exclusively to the proper interest of the person in
whose name it is made” cannot be validly seized upon to hold that the same
includes the beneficiary. The word “interest” highly suggests that the provision
refers only to the “insured” and not the beneficiary, since a contract of insurance
is personal in character. Otherwise, the prohibitory laws against illicit
relationships especially on property and descent will be rendered nugatory, as
the same could easily be circumvented by modes of insurance.

Rather, on matters not otherwise specifically provided for by the Insurance Law,
the contract of life insurance is governed by general rules of civil law. Under
Article 2012 of the Civil Code, “any person who is forbidden from receiving any
donation under Article 739 cannot be named beneficiary of a life insurance policy
by the person who cannot make a donation to him.” Common-law spouses are
definitely barred from receiving donations from each other. Life insurance policy
is no different from civil donation as far as beneficiary is concerned. Both are
founded on liberality.

2. NO. Conviction for adultery or concubinage for those barred from receiving
donations or life insurance not required as only preponderance of evidence is
necessary. The Court does not think that a conviction is exacted before the
disabilities mentioned in Article 739 may effectuate. More specifically, with regard
to the disability on “persons who were guilty of adultery or concubinage at the
time of donation,” xxx The underscored clause neatly conveys that no criminal
conviction for the disqualifying offense is a condition precedent. In fact, it cannot
even be gleaned from the aforequoted provision that a criminal prosecution is
needed. On the contrary, the law plainly states that the guilt of the party may be
proved “in the same action” for declaration of nullity of donation. And, it would be
sufficient if evidence preponderates upon the guilty of the consort for the offense
indicated. The quantum of proof in criminal cases is not demanded.

In the case, the requisite proof of common-law relationship between insured and
the beneficiary has been conveniently supplied by the stipulations between the
parties in the pre-trial conference of the case. These are considered judicial
admissions, of which judgment may be validly rendered without rigors of trial to
prove illicit relationship.

Albert G. Cong

GREAT PACIFIC LIFE ASSURANCE COMPANY vs. COURT OF APPEALS


89 SCRA 542 (1979)

FACTS:
Ngo Hing filed an application with the Great Pacific Life Assurance Company
(Grepalife) for a twenty-year endowment policy in the amount of P50,000.00 on the life
of his one-year old daughter Helen Go. Upon the payment of the insurance premium,
the binding deposit receipt was issued to Ngo Hing. Lapulapu Mondragon, the Cebu
City Branch Manager of Grepalife, later received a letter from Grepalife disapproving the
insurance application. The letter stated that the said life insurance application for 20-
year endowment plan is not available for minors below seven years old, but proposed
an alternative plan. The non-acceptance of the insurance plan by Grepalife was
allegedly not communicated to Ngo Hing by Mondragon, who instead, wrote back to
Grepalife again strongly recommending the approval of application.
.
Go died of influenza. Ngo Hing sought the payment of the proceeds of the
insurance, but having failed in his effort, he filed the action for the recovery before the
CFI of Cebu, which rendered an adverse decision against him.

ISSUE:

Does the binding deposit receipt constitute a temporary contract of life


insurance?

RULING:

No. The binding receipt in question was merely an acknowledgement on behalf


of the company, that the latter’s branch office had received from the applicant, the
insurance premium and had accepted the application subject for processing by the
insurance company, and that the latter will either approve or reject the same on the
basis of whether or not the applicant is insurable on standard rates.

Since Grepalife disapproved the insurance application of Ngo Hing, the binding
deposit receipt had never become in force at any time. A binding receipt is manifestly
merely conditional and does not insure outright. Where an agreement is made between
the applicant and the agent, no liability shall attach until the principal approves the risk
and a receipt is given by the agent. The acceptance is merely conditional, and is
subordinated to the act of the company in approving or rejecting the application. Thus
in life insurance, a binding slip or binding receipt does not insure by itself.

Also Ngo Hing had deliberately concealed the state of health of his daughter Go.
When he supplied data, he was fully aware that his one-year old daughter is typically a
mongoloid child. He withheld the fact material to the risk insured. The concealment
entitles the insurer to rescind the contract of insurance.

Rogaciano A. Quico III

TAN VS. CA
174 SCRA 403
FACTS:

Tan Lee Siong was issued a policy by Philamlife on Nov. 6, 1973. On Aprl 26,
1975, Tan died of hepatoma. His beneficiaries then filed a claim with Philamlife for the
proceeds of the insurance. Philamlife wrote the beneficiaries in Sep. 1975 denying their
claim and rescinding the contract on the ground of misrepresentation. The beneficiaries
contend that Philamlife can no longer rescind the contract on the ground of
misrepresentation as rescission must allegedly be done “during the lifetime of the
insured” within two years and prior to the commencement of the action following the
wording of Sec. 48, par. 2.

ISSUE:

Whether or not Philamlife can rescind the contract.

RULING:

YES. The so-called "incontestability clause" precludes the insurer from raising
the defenses of false representations or concealment of material facts insofar as health
and previous diseases are concerned if the insurance has been in force for at least two
years during the insured's lifetime. The phrase "during the lifetime" found in Section 48
simply means that the policy is no longer considered in force after the insured has died.
The key phrase in the second paragraph of Section 48 is "for a period of two years."

The policy was issued on November 6, 1973 and the insured died on April
26,1975. The policy was thus in force for a period of only one year and five months.
Considering that the insured died before the two-year period had lapsed, respondent
company is not, therefore, barred from proving that the policy is void ab initio by reason
of the insured's fraudulent concealment or misrepresentation.

The deceased was examined by Dr. Victoriano Lim and was found to be diabetic
and hypertensive. Another physician, Dr. Wenceslao Vitug, testified that the deceased
came to see him on December 14, 1973 for consultation and claimed to have been
diabetic for five years. Because of the concealment made by the deceased of his
consultations and treatments for hypertension, diabetes and liver disorders, respondent
company was thus misled into accepting the risk and approving his application.

Mary Christine Anthonette M. Salise-Punzalan

SUN INSURANCE LIFE OFFICE, Ltd. vs. Court of Appeals


211 SCRA 554/July 17, 1992
FACTS:

The petitioner issued personal life insurance policy to Felix Lim, Jr. with a face
value of Php200,000. Two months later, he was dead with a bullet wound in his head.
As beneficiary, his wife Nerissa Lim sought payment on the policy but her claim was
rejected. The petitioner agreed that there was no suicide and neither was there an
accident.

Pilar Nalagon, Lim’s secretary was the only eyewitness to his death. It happened
on October 6, 1982 at about 10 o’ clock in the evening after his mother’s birthday party.
According to Nalagon, Lim was in a happy mood (but not drunk) was playing with his
handgun, from which he had previously removed the magazine. As she watched
television, he stood in front of her and pointed the gun at her. She pushed it aside and
said it might be loaded. He assured her it was not loaded and pointed it in his temple.
The next moment there was an explosion and Lim was stumped to the floor. He was
dead before he fell.

Nerissa claimed as Felix’s beneficiary but Sun Insurance would not grant her
claim, saying that her husband’s death was not an accident.

Nerissa sued Sun Insurance and won the case. Sun Insurance was ordered to
pay her Php200,000 representing the face value of the claim along with moral,
exemplary and compensatory damages and attorney’s fees. The decision was affirmed
by the Court of Appeals.

Petitioner’s Claim

Sun Insurance cites one of the four exceptions in the contract of insurance which
includes bodily injury consequent upon the insured person attempting to commit suicide
or willfully exposing himself to needless peril in an attempt to save a human life.

The mere act of pointing the gun to his temple showed that Felix’ willfully
exposed himself to danger because a gun should always be handled with caution.

Respondent’s Claim

Felix believed the gun to be safe because he had removed the magazine. He
repeatedly assured his secretary that the gun was not loaded.

ISSUES:

1. Whether or not Felix Lim’s death was an accident, thus making his widow
Nerissa liable to claim the accident insurance.
2. Whether or not the award of damages to Nerissa Lim was justified.

RULING:

1. YES, Felix Lim’s death was an accident. There is no accident when a deliberate
act is performed unless some additional, unexpected, independent and
unforeseen happening occurs which produces or brings their injury or death.

An accident has been defined to be that which happens by chance of fortuitously


without intention or design and which is unexpected, unusual and foreseen. It an
event that takes pace without one’s foresight or expectations- an event that
proceeds from an unknown cause or is an unusual effect of a known case and
therefore not expected. It happens without any human agency, an event which,
under the circumstances, is unusual to and not expected by the person to whom
it happens.

The firing of the gun was deemed to be the unexpected and independent and
unforeseen occurrence that led to the insured person’s death.

There was no willful exposure to needless peril for the part of Felix. Suicide and
exposure to needless peril are similar in the sense that both signify disregard for
one’s life. Suicide imparts a positive act of ending one’s life whereas the latter
indicates recklessness that is almost suicidal in intent.

Accident insurance policies were never meant to reward the insured for his
tendency to show off or for his miscalculations. They were intended to provide for
contingencies.

Lim was unquestionably negligent but it should not prevent his widow from
recovering from the insurance policy he obtained precisely against accident.
Insurance contracts are, as a rule, supposed to be interpreted liberally in favor of
the assured.

2. NO, the claim for damages should not be granted for being unjust. A person may
be made liable to the payment of moral damages if his act is wrongful. The
adverse result of an action does not per se make the act wrongful and subject
the act or to the payment of moral damages.

Petitioner was acting in good faith when it resisted the private respondent’s claim
on the ground that the death of the insured was covered by the exception.

The issue was debatable and was clearly not raised only for the purpose of
evading a legitimate obligation.

You might also like